Download as pdf or txt
Download as pdf or txt
You are on page 1of 48

Prelims Wallah

Test
Test-4 of 38
(Geography and Disaster Management NCERT)

TARGET 2024

SCAN HERE FOR TEST SERIES ACCESS


1

Prelims Wallah Test 2024


Test 4: Geography and Disaster Management NCERT Test
1. The Demographic Transition Theory can be 3. Consider the following pairs of Tectonic
used to predict and describe the future plates and their locations:
population of any area as the society
Tectonic Plates Located between
progresses from rural, agrarian and illiterate
1. Coco plate : Philippines and New
to urban, industrial and literate society.
Guinea
Which of the following is correct about an
2. Nazca plate : Central America and
area, according to the above theory, as the
Pacific Plate
area undergoes the above mentioned
3. Caroline plate : South America and
transition?
Pacific Plate
(a) The Population of the region changes
Which of the pairs given above is/are
from high births and high deaths to low
correctly matched?
births and low deaths
(b) The Population of the region changes (a) 1 only
from low births and high deaths to low (b) 2 and 3 only
births and low deaths (c) 1 and 3 only
(c) The Population of the region changes (d) None of the above
from high births and low deaths to low
births and high deaths 4. Consider the following with reference to the
(d) The Population of the region changes resolutions adopted as part of the Yokohama
from low births and low deaths to high Strategy and Plan of Action for a Safer
births and high deaths. World:
1. It is the responsibility of developed
2. Physical geography includes the study of countries to protect the citizens of
which of the following? developing countries from natural
1. Landforms disasters.
2. Drainage 2. It will promote and strengthen sub-
3. Weather and Climate regional, regional and international
4. Temperature and pressure cooperation in mobilisation of resources.
5. Agriculture Which of the statements given above is/are
6. Oceans and lakes correct?
Select the correct answer using the codes (a) 1 only
given below: (b) 2 only
(a) 2, 3, 4, 5 and 6 only (c) Both 1 and 2
(b) 2, 5 and 6 only (d) Neither 1 nor 2
(c) 3, 4 and 5 only
(d) 1, 2, 3, 4 and 6 only 5. Which of the following statements is
incorrect?
(a) Water is generally brackish in Lagoons
and backwaters.
2

(b) The groundwater utilisation is very high How many of the above factors are
in the states of Punjab, Haryana, responsible for or affect the general
Rajasthan, and Tamil Nadu circulation of the atmosphere?
(c) Agriculture accounts for the highest (a) Only one
usage of groundwater in India (b) Only two
(d) Precipitation in India has temporal (c) Only three
variations but no spatial variation (d) All four

6. Which of the following factors are 9. Consider the following atmospheric


responsible for pedogenesis? conditions:
1. Climate 1. Large and continuous supply of warm
2. Biotic organisms and moist air
3. Time 2. Strong Coriolis force
4. Parent rock 3. Unstable condition which induces the
Select the correct answer using the code formation of local disturbances
given below: 4. Presence of strong vertical wind wedge
(a) 1 and 2 only How many of the above are initial conditions
(b) 1, 2 and 3 only required for the formation of a Tropical
(c) 1, 3, and 4 only Cyclone?
(d) 1, 2, 3 and 4 (a) Only one
(b) Only two
7. Consider the following statements with (c) Only three
reference to Tsunamis: (d) All four
1. The high vertical waves in seas and
oceans caused by earthquakes and 10. Consider the following statements with
volcanism are called tsunamis. respect to the Indian tectonic plate:
1. The subduction zone along the
2. The speed of tsunami waves in the seas
Himalayas forms the northern plate
and oceans varies with the depth of water.
boundary in form of continent-continent
Which of the statements given above is/are
convergence.
correct?
2. The Himalayas are older than Deccan
(a) 1 only
traps.
(b) 2 only
3. During the formation of the Deccan
(c) Both 1 and 2
traps, the Indian plate was close to the
(d) Neither 1 nor 2 equator.
How many of the above statements is/are
8. Consider the following: correct?
1. Latitudinal variation in insolation
(a) Only one
2. Existence of pressure belts (b) Only two
3. Distribution of continents and oceans (c) Only three
(d) None
4. Rotation of the earth.
3

11. With respect to the Slash and Burn (b) Both Statement-I and Statement-II are
Agriculture, consider the following pairs: correct and Statement-II is not the correct
Regional name Region explanation for Statement-I
1. Milpa : Mexico (c) Statement-I is correct but Statement-II is
2. Conuco : Venezuela incorrect
3. Ray : Central Africa (d) Statement-I is incorrect but Statement-II
4. Ladang : Indonesia is correct
How many of the pairs given above are
correctly matched? 14. Consider the following statements:
(a) Onle one pair 1. The longitudinal extent of India’s
(b) Only two pairs mainland is more than its latitudinal
(c) Only three pairs extent in kilometres.
(d) All four pairs 2. India's latitudinal extent far exceeds its
longitudinal extent in degrees.
12. Consider the following events regarding the 3. In terms of area, India is the seventh
evolution of the Earth’s atmosphere: largest country in the world.
1. The loss of primordial atmosphere
through the impact of solar wind. How many of the above statements is/are
incorrect?
2. Gases and water vapour were released
(a) Only one
from the interior of the solid earth.
(b) Only two
3. Oxygen accumulates in the atmosphere.
(c) All three
Which of the following is the correct
(d) None
chronological sequence of the above events?
(a) 1-3-2 15. With reference to the Endogenic geomorphic
(b) 2-3-1 processes, consider the following
(c) 3-1-2 statements:
(d) 1-2-3 1. They are mainly land-wearing forces
originating from the interiors of the earth.
13. Consider the following statements: 2. The action of these forces is not uniform,
Statement-I: With the increase in distance which makes the tectonically active
from the sea, the force of the cyclone crustal surface uneven.
decreases. Which of the statements given above is/are
Statement-II: The energy of the tropical correct?
cyclone comes from the latent heat released (a) 1 only
by the warm moist air. (b) 2 only
Which one of the following is correct in (c) Both 1 and 2
respect of the above statements?
(d) Neither 1 nor 2
(a) Both Statement-I and Statement-II are
correct and Statement-II is the correct
explanation for Statement-I
4

16. Consider the following statements: (a) Both Statement-I and Statement-II are
1. Rice and sugarcane are water-intensive correct and Statement-II is the correct
crops. explanation for Statement-I.
2. In the states of Punjab, Haryana, and (b) Both Statement-I and Statement-II are
Uttar Pradesh, irrigation covers around correct and Statement-II is not the correct
50% of the net sown area. explanation for Statement-I.
Which of the statements given above is/are (c) Statement-I is correct but Statement-II is
correct? incorrect.
(a) 1 only (d) Statement-I is incorrect but Statement-II
(b) 2 only is correct.
(c) Both 1 and 2
(d) Neither 1 nor 2 19. Consider the following statements with
reference to droughts:
17. With reference to the origin of our Universe 1. Meteorological Drought is a condition
and the Solar System, consider the following when there is a prolonged period of
pairs: inadequate rainfall.
Idea Proposition
2. Agricultural drought is also known as soil
1. Nebular The planets were formed moisture drought.
Hypothesis from a cloud of
material associated with 3. Hydrological Drought results when the
the youthful sun. availability of water in storages such as
aquifers falls below what the
2. Big Bang The universe is
precipitation can replenish.
Theory continuously expanding.

E
4. Ecological Drought is when the

IN
3. Binary Planets were formed after productivity of a natural ecosystem
Theory the collision of the sun
with another star. increases.

L
How many of the pairs given above are How many of the statements given above
correct?
(a) Only one
is/are correct?
N
.O
(a) Only one
(b) Only two
(b) Only two
(c) All three
L

(d) None (c) Only three


IA

(d) All four


18. Consider the following statements:
20. Which of the following are endogenic
R

Statement-I: The sun rises in Dibrugarh,


Assam, around two hours earlier than geomorphic processes?
E

Jaisalmer, Rajasthan. 1. Tectonic Plate Movement


T

Statement-II: There is a variation of around 2. Weathering


30 degrees between the extremes of 3. Volcanism
A

longitudes in India. 4. Karst Processes


M

Which of the following is correct with respect


5. Diastrophism
to the above statements?
C
S
P
U

https://t.me/visioniastestseries2024
5

Select the correct answer using the codes 23. In the context of Disaster Management
given below: studies, consider the following statements:
(a) 1 and 3 only 1. Hazard is the extent to which a
(b) 1, 3 and 5 only community, service, or geographic area
(c) 1, 3, 4 and 5 only is likely to get disrupted.
(d) 1, 2, 3, 4 and 5 2. Vulnerability means an event that has the
potential to cause injury to life or damage
21. Which of the following are examples of Zaid to property or the environment.
crops? Which of the statements given above is/are
1. Watermelon correct?
2. Maize (a) 1 only
3. Cucumbers
(b) 2 only
4. Groundnut
Select the correct answer using the codes (c) Both 1 and 2
given below: (d) Neither 1 nor 2
(a) 1 and 2 only
(b) 1 and 3 only 24. Which one of the following statements is
(c) 2, 3 and 4 only incorrect regarding ‘Dew point’?
(d) 1, 3 and 4 only (a) It is the temperature at which saturation
occurs in a given sample of air.
22. With reference to the planets in our solar (b) Dew is formed only when the dew point
system, consider the following statements: is below the freezing point.
Statement-I: The terrestrial planets are (c) Condensation can occur when the dew
point is above the freezing point.

E
smaller than the Jovian planets.
Statement-II: The lower gravity of the (d) The dew point occurs when Relative

IN
planetesimals which formed the terrestrial Humidity is 100%.
planets could not hold the abundant hydrogen

L
and helium. 25. Consider the following:
1. Faulting of the seafloor
N
Which one of the following is correct with
respect to the above statements? 2. Landslide
.O
(a) Both Statement-I and Statement-II are 3. Volcanic eruption
correct and Statement-II is the correct 4. Large meteoroid impact
L

explanation for Statement-I How many of the above can cause Tsunami?
(a) Only one
IA

(b) Both Statement-I and Statement-II are


correct and Statement-II is not the correct (b) Only two
explanation for Statement-I (c) Only three
R

(c) Statement-I is correct but Statement-II is (d) All four


E

incorrect
T

(d) Statement-I is incorrect but Statement-II


is correct
A
M
C
S
P
U

https://t.me/visioniastestseries2024
6

26. Consider the following statements: 29. Consider the following pairs regarding the
1. The atmosphere's density increases with names of cyclone:
increasing altitude. Regional name Associated Region
2. Higher concentration of dust particles is 1. Hurricane China
found in subtropical and temperate 2. Typhoons North America
regions compared to equatorial and polar 3. Willie-Willie South America
regions. 4. Tornado Australia
Which of the statements given above is/are How many of the pairs given above are
correct? correctly matched?
(a) 1 only (a) Only two
(b) 2 only (b) Only three
(c) Both 1 and 2 (c) All four
(d) Neither 1 nor 2 (d) None

27. Consider the following:


30. It is the natural process of large-scale
1. Coastal belt plantation
deformation of the Earth's crust that results in
2. Hazard Mapping
the formation of continents and ocean basins,
3. Land Use Control
mountain systems, plateaus, rift valleys, and
4. Engineered structures
other features through mechanisms like
How many of the above techniques can be
orogenesis, epeirogenesis, plate tectonics etc.
used to reduce the impact of a cyclone?
In the context of geomorphic processes,
(a) Only one which among the following processes is
(b) Only two

E
described by the above statement?
(c) Only three (a) Volcanism

IN
(d) All four (b) Diastrophism
(c) Weathering

L
28. Which among the following are formed by (d) Gradation
the condensation of water vapour present in
the atmosphere? N
31. Consider the following statements with
.O
1. Fog reference to Rabi crops:
2. Snow 1. They are sown in monsoon and harvested
L

3. Frost in October.
IA

4. Clouds 2. Western temperate cyclones help in the


5. Mist success of these crops.
6. Smog Which of the statements given above is/are
R

Select the correct answer using the code correct?


E

given below: (a) 1 only


(b) 2 only
T

(a) 1, 3, 4, and 6 only


(c) Both 1 and 2
A

(b) 1, 2, 3, 4, and 6 only


(d) Neither 1 nor 2
(c) 1, 2, 3, and 5 only
M

(d) 1, 2, 3, 4, 5, and 6
C
S
P
U

https://t.me/visioniastestseries2024
7

32. The term “Singularity” is often used in the 3. Bhutan : Assam and Arunachal
context of which of the following? Pradesh
(a) The Origin and evolution of life on Earth 4. China : Uttar Pradesh and
(b) Ocean exploration mission Himachal Pradesh.
(c) Study of solar and lunar eclipses Which of the above given pair(s) is/are
(d) Understanding the origins of the correctly matched?
universe. (a) 2 only
(b) 1, 2 and 3 only
33. Consider the following statements regarding (c) 1 and 4 only
the location of Manipur: (d) 1 and 3 only
1. Manipur is bordered by Nagaland to the
North, Tripura to the South and 36. Consider the following statements with
Myanmar to the East. reference to the troposphere:
2. The tropic of Cancer passes through 1. The average height of the troposphere is
Imphal, the capital city of Manipur. the same from the equator to the poles.
3. Manipur shares its borders with the 2. All changes in climate and weather take
maximum number of states in the North place outside this layer.
East. 3. The temperature in this layer decreases at
How many of the statements given above a rate of one degree Celsius per 165 m of
is/are incorrect? height.
(a) Only one How many of the statements given above
(b) Only two is/are correct?
(c) All three (a) Only one

E
(d) None (b) Only two
(c) All three

IN
34. With reference to Relief rainfall, consider the (d) None
following statements:

L
1. This type of rainfall is closely associated 37. Consider the following:
with warm and cold fronts.
2. The leeward slopes receive greater
1. Desiltation of the river bed
2. Landslides N
.O
rainfall. 3. Earthquakes
Which of the statements given above is/are How many of the above can lead to floods?
L

incorrect? (a) Only one


IA

(a) 1 only (b) Only two


(b) 2 only (c) All three
R

(c) Both 1 and 2 (d) None


(d) Neither 1 nor 2 38. Which one among the following best
E

describes the Savannah type of climate?


T

35. Consider the following pairs: (a) Rainfall throughout the year.
Countries Bordering States
A

(b) Rainfall only in Winter.


1. Nepal : Sikkim and West
(c) An extremely short dry season.
M

Bengal
(d) A short wet season and a long dry season.
2. Bangladesh : Tripura and Manipur
C
S
P
U

https://t.me/visioniastestseries2024
8

39. Consider the following cities in India: 42. With reference to the types of volcanoes,
1. Lucknow consider the following pairs:
2. Ranchi Volcanoes Features
1. Composite Pyroclastic
3. Raipur
Volcanoes material
4. Bhubaneshwar
accumulates near
5. Hyderabad the vent opening
Which of the above given cities lies to the 2. Mid-Oceanic Basaltic lava is a
west of the Standard Meridian of India? Ridge major component
(a) 1, 3 and 5 only Volcanoes of these volcanoes
(b) 2 and 5 only 3. Shield Hawaiian
(c) 1, 2, 3 and 5 only Volcanoes volcanoes are
(d) 3 and 5 only examples of Shield
Volcanoes
40. In the context of exogenic geomorphic 4. Dormant Not erupted in a
processes, consider the following pairs: Volcanoes very long time
Denudational processes Driving force How many of the above pairs are correctly
1. Weathering : Chemical action matched?
2. Mass Movement : Kinetic Energy (a) Only one
3. Erosion : Gravitational (b) Only two
force (c) Only three
Which of the pairs given above is/are (d) All four
correctly matched?

E
(a) 1 only 43. Consider the following statements regarding
the location of India:

IN
(b) 2 only
(c) 1 and 3 only 1. The Southern part lies within the tropics,
(d) 1, 2 and 3 while the Northern part lies in the warm

L
temperate zone.
41. Consider the following statements:
N
2. The latitudinal extent of India has an
.O
1. About one-fourth of the total cropped impact on the natural vegetation and
area in India is under rice cultivation. climate variations.
2. Punjab and Haryana were chosen Which of the above statements is/are
L

initially for the Green Revolution as they incorrect?


IA

were traditional rice-growing areas. (a) 1 only


Which of the statements given above is/are (b) 2 only
R

correct? (c) Both 1 and 2


(a) 1 only (d) Neither 1 nor 2
E

(b) 2 only 44. With reference to Fogs, consider the


T

(c) Both 1 and 2 following statements:


A

(d) Neither 1 nor 2 1. Fogs are common where warm and cold
air currents come into contact with each
M

other.
2. Fog contains more moisture than mist.
C
S
P
U

https://t.me/visioniastestseries2024
9

Which of the statements given above is/are 4. Annapurna


correct? 5. Kamet
(a) 1 only Select the correct answer using the code
(b) 2 only given below:
(c) Both 1 and 2 (a) 1 and 4 only
(d) Neither 1 nor 2 (b) 3 and 5 only
(c) 2, 3 and 4 only
45. Which among the following is the minimum (d) 1, 2, 3 and 4 only
number of states one has to cross while
moving from Imphal to Shimla, including 48. Which of the following factors can influence
origin and destination states? the movement of ocean water?
(a) 5 States 1. Temperature
(b) 6 States 2. Gravity
(c) 8 States
3. Coriolis force
(d) 4 States
4. Obstruction due to land
46. With respect to the insolation, received at the 5. Wind
top of Earth’s atmosphere, consider the Select the correct answer using the code
following: given below:
Statement-I: Insolation received is maximum (a) 1 and 2 only
on the 4th of July. (b) 3, 4 and 5 only
Statement-II: Insolation received decreases (c) 2, 3, 4 and 5 only
with an increase in distance from the sun.

E
(d) 1, 2, 3, 4 and 5
Which one of the following is correct about

IN
the above statements? 49. ‘It originates from a glacier near Bokhar
(a) Both Statement-I and Statement-II are Chu and enters Pakistan near Chilas in the

L
correct and Statement-II is the correct Dardistan region and is known as 'Singi

N
explanation for Statement-I. Khamban' in Tibet. It also receives Panjnad
(b) Both Statement-I and Statement-II are near the Mithankot region.”
.O
correct and Statement-II is not the correct Which of the following rivers is best
explanation for Statement-I. described by the passage given above?
L

(a) Chenab
(c) Statement-I is correct but Statement-II is
(b) Indus
IA

incorrect.
(c) Jhelum
(d) Statement-I is incorrect but Statement-II
(d) Ravi
R

is correct.
E

50. Which of the following is/are


47. Which of the following Himalayan peaks are
characteristic(s) of the young stage of
T

located in India?
running water regime?
A

1. Makalu
1. Well-integrated flow with numerous
2. Dhaulagiri
M

tributaries and well-developed


3. Nanga Parbat
floodplains.
C
S
P
U

https://t.me/visioniastestseries2024
10

2. Waterfalls and rapids may exist where Which one of the following is correct in
streams encounter hard rocks. respect of the above statements?
(a) Both Statement-I and Statement-II are
3. Young rivers of the Himalayas are often
correct and Statement-II is the correct
dynamic and play a significant role in
explanation for Statement-I
shaping the landscape
(b) Both Statement-I and Statement-II are
Select the correct answer using the code
correct and Statement-II is not the correct
given below:
explanation for Statement-I
(a) 1 only
(c) Statement-I is correct but Statement-II is
(b) 1 and 2 only
incorrect
(c) 2 and 3 only
(d) Statement-I is incorrect but Statement-II
(d) 1, 2 and 3
is correct

51. Consider the following statements:


Statement-I: Tertiary activities are related to
53. Consider the following statements:
the service sector.
1. Dal Lake is located between the Ladakh
Statement-II: In tertiary activities, the
and the Karakoram range.
expertise provided by services relies more
2. Karewa formations aid in the cultivation
heavily on the production techniques,
of Zafran.
machinery and factory processes.
Which of the statements given above is/are
Which one of the following is correct in
correct?
respect of the above statements?
(a) 1 only
(a) Both Statement-I and Statement-II are
correct and Statement-II is the correct (b) 2 only

E
explanation for Statement-I (c) Both 1 and 2

IN
(b) Both Statement-I and Statement-II are (d) Neither 1 nor 2
correct and Statement-II is not the correct

L
explanation for Statement-I
(c) Statement-I is correct but Statement-II is 54. Which one of the following statements best
incorrect describes an Oasis? N
.O
(d) Statement-I is incorrect but Statement-II (a) A region within the desert where
is correct significant animal husbandry takes place.
L

(b) A region created in low-lying areas of the


52. Consider the following statements about
IA

desert where subterranean water emerges


volcanoes: to the surface, resulting in fertile regions.
Statement-I: The extent and nature of the (c) It is a fertile area located near a desert,
R

Caldera’s explosiveness gives an idea about affected by the desert's encroachment.


E

the proximity of the Magma Chamber. (d) The gravel plains and exposed rocky
T

Statement-II: Composite Volcanoes are plateaus within the desert's territory.


A

characterised by the eruption of cooler and


more viscous lavas than Shield volcanoes.
M
C
S
P
U

https://t.me/visioniastestseries2024
11

55. Consider the following statements with (b) The agriculture of the country is entirely
reference to the tributaries of the Indus River: dependent on rainfall during the
1. Ravi flows through the Chamba Valley of monsoon season.
Himachal Pradesh. (c) Winter rainfall by temperate cyclones in
2. Beas flows through the Kullu Valley, and north India is highly beneficial for rabi
forms gorges in the Dhauladhar range. crops
3. Sutlej is an antecedent river known as (d) Regional variations in rainfall result in
Langchen Khambab in Tibet. the mono cropping pattern in the country.
Which of the statements given above are
correct? 58. With reference to waves, consider the
(a) 1 and 2 only following pairs:
(b) 1 and 3 only Terms Description
(c) 2 and 3 only 1. Wave : It is one-half of
(d) 1, 2 and 3 frequency the wave height.
2. Wave crest : The lowest point
56. Consider the following statements with of a wave.
respect to variance in insolation on the 3. Wavelength : The rate at
Earth’s surface: which the wave
Statement-I: Insolation received increases moves through
as we move towards poles. the water
Statement-II: Insolation received is directly How many of the above pairs are incorrectly
proportional to the angle of rays with respect matched?
to Earth’s surface. (a) Only one

E
Which one of the following is correct about (b) Only two
the above statements?

IN
(c) All three
(a) Both Statement-I and Statement-II are (d) None
correct and Statement-II is the correct

L
explanation for Statement-I. 59. Which one of the following best describes the
(b) Both Statement-I and Statement-II are term ‘Bugyals’?
N
(a) Below sea level rice cultivation system in
.O
correct and Statement-II is not the correct
explanation for Statement-I. Kerala.
(b) Large pieces of lands that are used as salt
L

(c) Statement-I is correct but Statement-II is


incorrect. pans in Rann of Kutch.
IA

(d) Statement-I is incorrect but Statement-II (c) Hill slopes in Darjeeling used for tea
is correct. cultivation.
R

(d) Alpine meadows in the higher reaches of


Uttarakhand.
E

57. Which of the following statements is correct


with reference to India?
T

(a) The highest rainfall occurs along the


A

eastern coast as well as in the Himalayan


M

region.
C
S
P
U

https://t.me/visioniastestseries2024
12

60. Consider the following statements regarding 63. Consider the following tributaries of the
gorges and canyons: Ganga River:
1. A gorge is a deep valley with very steep 1. Gomti
to straight sides, and the canyon is 2. Ghaghra
characterised by steep step-like side 3. Gandak
slopes. 4. Kosi
2. A gorge is significantly wider at its top 5. Son
than at its bottom, and a canyon is almost 6. Yamuna
equal in width at its top as well as its 7. Damodar
bottom. How many of the above join the Ganga on its
Which of the statements given above is/are right bank?
correct? (a) Only three
(a) 1 only (b) Only four
(b) 2 only (c) Only five
(c) Both 1 and 2 (d) Only six
(d) Neither 1 nor 2
64. Which one of the following statements is
61. Consider the following activities: incorrect with respect to Spring Tides?
1. Street peddling (a) These tides occur during the full moon
2. Sell on Handcarts or trucks and new moon periods.
3. Door-to-door selling (b) These tides occur when the sun, the earth
4. Automatic vending machines and the moon are in a straight line.
5. Internet selling (c) These tides are caused by the indentions

E
How many of the above are examples of non- in the coastlines.

IN
store retail trading? (d) There is a seven-day interval between the
(a) Only two spring tides and neap tides.

L
(b) Only three
(c) Only four 65. Consider the following statements:
(d) All five N
1. The altitudinal variations are greater in
.O
the western half of the Himalayas than
62. Which one of the following statements is those in the eastern half.
L

correct about Dykes?


2. Shiwaliks are the most continuous range
(a) These are common in the plains of north
IA

in the Himalayas.
India Which of the statements given above is/are
(b) They are formed when the lava solidifies correct?
R

in a horizontal fashion (a) 1 only


E

(c) Dykes can be igneous or sedimentary in (b) 2 only


T

origin
(c) Both 1 and 2
(d) They are similar in appearance to shield
A

(d) Neither 1 nor 2


volcanoes
M
C
S
P
U

https://t.me/visioniastestseries2024
13

66. Consider the following statements: 2. The diurnal range of temperature in the
Statement-I: The earth’s surface receives Amazon basin is negligible.
most of its energy in radiations of short 3. Rainfall occurs nearly every day, as
wavelengths. thundershowers in the afternoon.
Statement-II: The energy received from the Select the correct answer using the code
Sun by the Earth is known as insolation. given below:
Which one of the following is correct in (a) 1 only
respect of the above statements? (b) 1 and 3 only
(a) Both Statement-I and Statement-II are (c) 2 and 3 only
correct and Statement-II is the correct (d) 1, 2 and 3
explanation for Statement-I
(b) Both Statement-I and Statement-II are 69. With reference to the peninsular drainage
correct and Statement-II is not the correct system of India, consider the following
explanation for Statement-I statements:
(c) Statement-I is correct but Statement-II is 1. Peninsular drainage system is younger
incorrect than the Himalayan drainage system.
(d) Statement-I is incorrect but Statement-II 2. The Chambal, the Sind, the Betwa, the
is correct Ken, and the Son, originating in the
Peninsula, belong to the Ganga river
67. Consider the following pairs with reference system.
to local storms in the hot weather season in 3. Mahi is the only river in India that cuts
India: tropic of cancer twice.
Name State How many of the above statements are

E
1. Mango Shower : Kerala correct?
2. Blossom : Maharashtra

IN
(a) Only one
Shower (b) Only two
3. Nor Westers : Bengal

L
(c) All three
4. Loo : Northern
(d) None
plains
How many of the pairs given above are N
.O
70. With reference to Karst topography, consider
correctly matched?
the following statements:
(a) Only one pair
L

1. It is mainly produced by the action of


(b) Only two pairs
groundwater.
IA

(c) Only three pairs


2. It is a landform formed only by
(d) None of the pairs erosional processes.
R

Which of the Statements given above is/are


68. Which of the following statements(s) is/are correct?
E

correct regarding the climate in the Amazon (a) 1 only


T

basin? (b) 2 only


1. The Amazon Basin lies along the equator
A

(c) Both 1 and 2


and is known for its consistently warm (d) Neither 1 nor 2
M

and moist weather throughout the year.


C
S
P
U

https://t.me/visioniastestseries2024
14

71. Consider the following pairs: Which of the above currents is/are cold ocean
Places Country currents?
1. Odessa Azerbaijan (a) 1 only
2. Baku Mongolia (b) 3 and 4 only
3. Ulan Bator Ukraine (c) 1, 3 and 4 only
4. Tashkent Uzbekistan (d) 1, 2, 3, and 4
How many of the pairs given above is/are
correctly matched? 75. With reference to differential heating of land
(a) Only one pair and sea, consider the following statements:
(b) Only two pairs 1. As compared to sea, land heats up or
cools down quickly.
(c) Only three pairs
2. Places situated near to sea experience less
(d) All four pairs
temperature variation.
Which of the statements given above is/are
72. Consider the following statements regarding
correct?
the Propagation of Earthquake Waves:
(a) 1 only
1. The motion of the media in P waves is (b) 2 only
perpendicular to the direction of wave (c) Both 1 and 2
propagation. (d) Neither 1 nor 2
2. S waves are slower than P waves.
3. P and S waves propagate in a solid 76. Consider the following statements:
medium. 1. Dendritic drainage is the most commonly
How many of the above statements are found drainage pattern in nature; it

E
correct? resembles a tree.
2. Centripetal rivers are rivers that flow

IN
(a) Only one
(b) Only two towards a central point, such as a lake or
depression.

L
(c) All three
Which of the above statements is/are correct?
(d) None
(a) 1 only
N
.O
73. Which one of the following states is also (b) 2 only
referred to as the Molassis basin? (c) Both 1 and 2
L

(a) Mizoram (d) Neither 1 nor 2


IA

(b) Nagaland
77. Which one among the following forest types
(c) Meghalaya
is characterised by the prevalence of Orchids
R

(d) Manipur and Bromeliads?


E

(a) Rain Forest


74. Consider the following ocean currents: (b) Savanah grasslands
T

1. Labrador Current (c) Marshy Forest


2. Gulf Stream
A

(d) Deciduous Forest


3. Falkland Current
M

4. Oyashio Current 78. Consider the following statements:


C

1. These zones experience foggy weather.


S
P
U

https://t.me/visioniastestseries2024
15

2. These zones provide the best routes for (a) 1 only


navigation. (b) 2 only
3. These zones are one of the best fishing (c) Both 1 and 2
grounds of the world. (d) Neither 1 nor 2
How many of the statements given above
is/are correct regarding the mixing zones of 82. Consider the following statements about the
warm and cold ocean currents? Earthquake Waves:
(a) Only one 1. Body and Surface waves both form
(b) Only two shadow zones.
(c) All three 2. The shadow zone of the S-wave is
(d) None smaller than that of the P-wave.
3. The shadow zones of P-wave and S-
79. Consider the following statements: wave overlap with one another.
1. Karbi-Anglong Pleateau and the How many of the above statements are
Meghalaya Plateau are extensions of the correct?
Peninsular Plateau.
(a) Only one
2. The valleys through which Tapi and
(b) Only two
Mahanadi flow have formed only as a
result of the erosional activities of these (c) All three
rivers. (d) None
Which of the statements given above is/are
correct? 83. Consider the following statements:
(a) 1 only 1. Easterly jet streams are responsible for

E
(b) 2 only the burst of monsoon in India.
(c) Both 1 and 2 2. The shift of the Intertropical convergence

IN
(d) Neither 1 nor 2 zone northwards causes the withdrawal
of the westerly jetstream from India.

L
80. ‘Cirque, Arete, Truncated Spurs, and Which of the statements given above is/are

N
Moraine’ are the landforms associated with incorrect?
which of the following? (a) 1 only
.O
(b) 2 only
(a) Groundwater
(c) Both 1 and 2
(b) River
L

(d) Neither 1 nor 2


(c) Waves
IA

(d) Glaciers 84. Consider the following:


1. The distortion of equatorial atmospheric
R

81. Consider the following statements with circulation


E

reference to the modes of transportation: 2. Irregularities in the evaporation of sea


1. All modes of transportation carry both water
T

passenger and freight. 3. Reduction in the amount of plankton in


A

2. Road transport is costlier for door-to- some areas.


door services over short distances.
M

Which of the above is/are the impacts of


Which of the statements given above is/are El-Nino?
C

correct?
S
P
U

https://t.me/visioniastestseries2024
16

(a) 1 and 3 only 3. The Coriolis force acts perpendicular to


(b) 2 and 3 only the pressure gradient force
(c) 1 and 2 only How many of the above statements is/are
(d) 1, 2 and 3 correct?
(a) Only one
85. Consider the following: (b) Only two
1. Pressure (c) All three
2. Friction from the land (d) None
3. Coriolis force
4. Gravitational force.
How many of the above factors affect the 89. Consider the following statements with
velocity and direction of the wind? reference to the rivers:
(a) Only one 1. Ephemeral rivers are those that have
(b) Only two water in them throughout the year.
(c) Only three 2. A river's catchment area refers to the
region from which it collects water.
(d) All four
3. A drainage basin is a region that is
drained by a river and its tributaries.
86. Consider the following organism:
4. A watershed is the boundary line that
1. Fungi separates one drainage basin from
2. Bacteria another.
3. Flagellates How many of the statements given above are
How many of the above are classified as correct?
decomposers? (a) Only one

E
(a) Only one (b) Only two
(b) Only two (c) Only three

IN
(c) All three (d) All four
(d) None

L
90. With reference to various types of landforms,
87. Which one of the following is the correct
sequence of the deposition of alluvium in
consider the following statements:
N
.O
1. Fjords are landforms found on low
the northern plains of India? sedimentary coasts.
(a) Bhabhar - Tarai - Bhangar - Khadar 2. Lagoons are former glacial valleys in
L

coastal areas.
(b) Bhangar - Khadar - Tarai - Bhabhar
IA

3. The west coast of India is a high, rocky,


(c) Tarai - Bhabhar - Khadar - Bhangar retreating coast, and the east coast of
(d) Khadar - Bhangar - Bhabhar - Tarai India is a low, sedimentary coast.
R

How many of the statements given above are


E

88. Consider the following statements with correct?


respect to the air pressure: (a) Only one
T

1. The pressure gradient is weak where the (b) Only two


A

isobars are close to each other. (c) All three


2. The frictional force is greatest at the (d) None
M

surface of the earth.


C
S
P
U

https://t.me/visioniastestseries2024
17

91. Consider the following statements with


reference to the Trans-Siberian railway line: 94. Consider the following statement:
1. It is the longest double-tracked and 1. The Arabian and the Bay of Bengal
electrified transcontinental railway in the drainage systems are separated by the
world. Eastern Ghats, the Delhi Ridge, and the
2. It runs across the Ural Mountains. Western Ghats.
Which of the statements given above is/are 2. The majority of peninsular rivers flow
correct? into the Bay of Bengal from their sources
(a) 1 only in the Eastern Ghats.
(b) 2 only Which of the statements given above is/are
correct?
(c) Both 1 and 2
(a) 1 only
(d) Neither 1 nor 2
(b) 2 only
(c) Both 1 and 2
92. With reference to the Earth’s Crust, consider
(d) Neither 1 nor 2
the following statements:
1. Volcanic eruptions are uncommon near 95. With reference to the atmospheric circulation
mid-oceanic ridges. of air, consider the following:
2. Oceanic crust rocks are much older than Statement-I: Wind blows perpendicular to
continental crust rocks. the isobars at the equator
3. The sediment deposits on the ocean floor Statement-II: Coriolis force is zero at the
are unexpectedly thin. equator
How many of the statements given above are Which one of the following is correct in

E
incorrect? respect of the above statements?

IN
(a) Only one (a) Both Statement-I and Statement-II are
(b) Only two correct and Statement-II is the correct
explanation for Statement-I

L
(c) All three
(b) Both Statement-I and Statement-II are
(d) None
N
correct and Statement-II is not the correct
.O
93. Consider the following functions: explanation for Statement-I
1. Conversion of inorganic into organic (c) Statement-I is correct but Statement-II is
incorrect
L

material
(d) Statement-I is incorrect but Statement-II
IA

2. Consumption of the autotrophs by


heterotrophs is correct
3. Decomposition and mineralisation of the
R

dead matter 96. Which of the following statements best


describes the term 'tillite'?
E

How many of the above are considered the


(a) A consolidated or indurated sedimentary
functions of an ecosystem?
T

rock formed by lithification of glacial till.


(a) Only one
A

(b) They are long, narrow and steep


(b) Only two depressions on the abyssal plain of the
M

(c) All three sea


(d) None
C
S
P
U

https://t.me/visioniastestseries2024
18

(c) They are deep concave gorges on the 99. With reference to the Continental Drift
continental shelf and slope. Hypothesis, polar fleeing forces are related to
(d) They are undulating regions lying which of the following?
between the foot of the continental rise (a) Revolution of the earth
and the mid-oceanic ridge. (b) Rotation of the earth
(c) Tilting of the earth on its own axis
97. Which of the following statements is correct
(d) The gravitational force of the Sun and the
regarding secondary productivity?
moon
(a) It is organic matter produced per unit area
over a time period by plants.
100. Consider the following pairs of sea routes in
(b) It is the rate of production of organic
the world:
matter during photosynthesis.
Sea Route Connects
(c) It is gross primary productivity minus
1. Suez Canal Pacific Ocean and
respiration loss.
Atlantic Ocean
(d) It is the rate of formation of new organic
2. Panama Canal Mediterranean Sea
matter by consumers.
and Red Sea
3. South Pacific Western Europe
98. Consider the following statements with
Sea route and Australia
reference to water vapour present in the
How many of the pairs given above are
atmosphere:
correctly matched?
1. The amount of water vapour present in
the atmosphere generally decreases with (a) Only one pair
an increase in altitude. (b) Only two pairs

E
2. A decrease in the amount of water vapour (c) All three pairs
in the air from the equator towards the

IN
(d) None of the above
pole is a general trend.
3. It acts like a blanket for the earth thus

L
preventing it from becoming too hot or
too cold.
How many of the statements given above N
.O
is/are correct?
(a) Only one
L

(b) Only two


IA

(c) All three


(d) None
R
E
T
A
M
C
S
P
U

https://t.me/visioniastestseries2024
1

Prelims Wallah Test 2024


Test 4: Geography and Disaster Management Synopsis
1. A 26. B 51. C 76. C
2. D 27. D 52. B 77. A
3. D 28. D 53. B 78. B
4. B 29. D 54. B 79. A
5. D 30. B 55. D 80. D
6. D 31. B 56. D 81. D
7. C 32. D 57. C 82. A
8. D 33. C 58. C 83. D
9. C 34. C 59. D 84. D
10. B 35. D 60. A 85. D
11. C 36. A 61. D 86. C
12. D 37. B 62. C 87. A
13. A 38. D 63. A 88. B
14. B 39. A 64. C 89. C
15. B 40. A 65. D 90. A
16. A 41. A 66. B 91. C
17. C 42. D 67. C 92. B
18. A 43. D 68. B 93. C
19. C 44. A 69. B 94. D
20. B 45. B 70. A 95. A
21. B 46. D 71. A 96. A
22. A 47. B 72. B 97. D
23. D 48. D 73. A 98. C

E
24. B 49. B 74. C 99. B

IN
25. D 50. C 75. C 100. A
L
N
.O
L
IA
R
E
T
A
M
C
S
P

https://t.me/visioniastestseries2024
2

1. Ans: A Climate factors, Temperature, Pressure,


Exp: Winds, Precipitation, Climatic types, etc.):
The demographic transition theory can be used Options 3 and 4 are correct.
to describe and predict the future population of Study of the Hydrosphere (such as Ocean,
any area. The theory says that the population Sea, Lake, Salinity, Flow, etc.): Option 6 is
of any region changes from high birth and correct
Study of Biospheres such as life forms,
high death to low birth and low death as
ecosystems, and other related phenomena.
society progresses from rural agrarian and
Agriculture and other economic activities of
illiterate to urban industrial and literate human beings are studied as a part of Human
society. These changes occur in three stages Geography. Option 5 is incorrect.
which are collectively known as demographic Reference: NCERT Class 12th, Fundamentals
cycle. of Physical Geography, Chapter 1
The three stages are:
1. The first stage has high fertility and 3. Ans: D
high mortality as people reproduce Explanation:
more to compensate for the death due Pair 1 is incorrectly matched: Cocos Plate:
Between Central America and the Pacific Plate
to epidemics and variable food supply.
Pair 2 is incorrectly matched: Nazca Plate:
The population growth is slow and Between South America and the Pacific Plate
most people are engaged in agriculture Pair 3 is incorrectly matched: Caroline Plate:
where large families are an asset. Life Between the Philippine and Indian plates
expectancy is low and people are (North of New Guinea)
mostly illiterate. The tectonic plate (also called the lithospheric
2. Fertility remains high in the beginning plate) is a massive, irregularly shaped slab of
solid rock, that comprises both continental and
of the second stage but declines with
oceanic lithosphere. As rigid units, plates move
time ; this is accompanied by reduced horizontally over the asthenosphere. A plate
mortality rate. Improvement in may be referred to as the continental plate or the
sanitation and health conditions lead to oceanic plate, depending on which of the two
occupies a larger portion of the plate. The

E
decline in mortality. Because of this
gap, net addition to the population is Pacific Plate is largely an oceanic plate,

IN
high. whereas the Eurasian Plate may be called a
continental plate. The theory of plate tectonics
3. In the third and last stage, both fertility proposes that the earth’s lithosphere is divided
L
and mortality decline considerably. into seven major and some minor plates:
N

The population is either stable or grows Major tectonic plates


slowly. The population becomes Antarctica and the surrounding oceanic plate
.O

urbanized, literate and has high North American plate


technical knowledge and deliberately South American plate
Pacific plate
L

controls the family size.


India-Australia-New Zealand plate
IA

Reference: NCERT, Class 12th, Fundamentals Africa with the eastern Atlantic floor plate
of Human Geography, Chapter 2 Eurasia and the adjacent oceanic plate
Minor tectonic plates
R

2. Ans: D Cocos Plate: Between Central America and the


Exp:
E

Pacific Plate
Physical geography is a branch of geography Nazca Plate: Between South America and the
T

that includes the study of: Pacific Plate


Study of the Lithosphere (including
A

Arabian plate: Mostly the Saudi Arabian


Landform, Drainage, Relief and landmass
M

Physiography): Options 1 and 2 are correct Philippine plate: Between the Asiatic and
Study of the Atmosphere (including Pacific plate
Composition, Structure, Elements, Weather and
C
S
P

https://t.me/visioniastestseries2024
3

Caroline Plate: Between the Philippine and states. Due to this, many lagoons and lakes have
Indian plates (North of New Guinea) formed. States like Kerala, Odisha and West
Fuji Plate: North-east of Australia. Bengal have vast surface water resources in
Aegean plate (Mediterranean region) these lagoons and lakes. Although water is
Caribbean plate generally brackish in these water bodies, it is
Juan de Fuca plate (between Pacific and North used for fishing and irrigating certain varieties
American plates) of paddy crops, coconut, etc.
Iranian plate Statement 2 is correct: The groundwater
Reference: NCERT, Class 11th, Fundamentals utilisation is very high in the states of
of Physical Geography, Chapter 4 Punjab, Haryana, Rajasthan, and Tamil
Nadu. However, there are States like
4. Ans: B Chhattisgarh, Odisha, Kerala, etc., which utilise
Exp: only a small proportion of their groundwater
The member states of the United Nations and potential. States like Gujarat, Uttar Pradesh,
other states met at the World Conference on Bihar, Tripura and Maharashtra are utilising
Natural Disaster Reduction in the city of their groundwater resources moderately.
Yokohama, Japan from May 23rd-27th,1994. It Statement 3 is correct: Agriculture accounts
accepted that disasters affected the poor and for most of the surface and groundwater
disadvantaged groups the worst, particularly in utilisation; it accounts for 89 per cent of the
the developing countries, which are ill- surface water and 92 per cent of the
equipped to cope with them. They adopted a groundwater utilisation. While the share of the
resolution later known as the Yokohama industrial sector is limited to 2 per cent of
Strategy and Plan of Action for a Safer surface water utilisation and 5 per cent of
World, which includes: groundwater, the share of the domestic sector is
● It will note that each country has the higher (9 per cent) in surface water utilisation
sovereign responsibility to protect its as compared to groundwater.
citizens from natural disasters. Statement 4 is incorrect: Precipitation in
Hence, Statement 1 is incorrect. India has very high spatial variation, and it is
● It will give priority attention to mainly concentrated in the Monsoon season.
developing countries, particularly the The precipitation is relatively high in the
least developed, land-locked countries catchment areas of the Ganga, the Brahmaputra

E
and small-island developing states. and the Barak rivers; these rivers, although

IN
● It will promote and strengthen sub- account for only about one-third of the total
regional, regional, and international area in the country, have 60 percent of the total
cooperation in activities to prevent, surface water resources.
L
reduce, and mitigate natural and other Reference: NCERT, Class 12th, India People
N

disasters, with particular emphasis on and Economy, Chapter 4


1. Human and institutional
.O

capacity-building and 6. Ans: D


strengthening Exp:
2. Technology sharing: the The process of formation of soil is called
L

collection, dissemination, and pedogenesis. It depends on several factors:


IA

utilisation of information 1. Climate: Option 1 is correct


3. Mobilisation of resources. 2. Bioactivity: Option 2 is correct
Hence, Statement 2 is 3. Time: Option 3 is correct
R

correct. 4. Parent material of the underlying rock:


E

Reference: NCERT, Class 11th, India Physical Option 4 is correct


Environment, Chapter 6
T
A

5. Ans: D
Exp:
M

Statement 1 is correct: India has a vast


coastline, and the coast is very indented in some
C
S
P

https://t.me/visioniastestseries2024
4

atmosphere also sets in motion the ocean water


circulation, which influences the earth’s
climate. The pattern of planetary winds depends
mainly on:
● Latitudinal variation in insolation.
Hence, Option 1 is correct.
● Existence of pressure belts. Hence,
Option 2 is correct.
● Migration of belts following the
apparent path of the Sun.
● Distribution of continents and oceans.
Fig: Factors responsible for Pedogenesis.
Hence, Option 3 is correct.
Reference: NCERT, Class 11th, Fundamentals
● Rotation of earth. Hence, Option 4 is
of Physical Geography, Chapter 1
correct.
7. Ans: C
Exp:
Statement 1 is correct: Earthquakes and
volcanic eruptions that cause the sea floor to
move abruptly, resulting in sudden
displacement of ocean water in the form of high
vertical waves are called tsunamis or seismic
sea waves. Normally, the seismic waves cause
only one instantaneous vertical wave, but, after
the initial disturbance, a series of afterwaves are
created in the water that oscillate between high
crest and low trough in order to restore the
water level. Fig: General circulation of the atmosphere.
Statement 2 is correct: The speed of tsunami Reference: NCERT, Class 11th, Fundamentals
waves in the seas and oceans varies with the of Physical Geography, Chapter 9
depth of water. In the deep ocean, tsunamis

E
can move as fast as a jet plane, over 500 mph, 9. Ans: C
and can cross entire oceans in less than a day. Exp:

IN
As the waves enter shallow water near land, Tropical cyclones are intense low-pressure
they slow to the speed of a car, approximately areas confined to the area lying between 30°N
L
20 or 30 mph. and 30°S latitudes, in the atmosphere around
which high-velocity winds blow. Initial
N

conditions required for the emergence of a


tropical cyclone are
.O

● Large and continuous supply of


warm and moist air that can release
L

enormous latent heat. Hence, Option 1


is correct
IA

● Strong Coriolis force that prevents the


filling of low pressure at the centre
R

(absence of Coriolis force near the


Reference: NCERT, Class 11th, India Physical equator prohibits the formation of
E

Environment, Chapter 6 tropical cyclones between 0°-5°


latitude). Hence, Option 2 is correct
T

8. Ans: D ● Unstable condition (low pressure)


A

Exp: through the troposphere creates local


The pattern of the movement of the planetary disturbances around which a cyclone
M

winds is called the general circulation of the develops. Hence, Option 3 is correct
atmosphere. The general circulation of the
C
S
P

https://t.me/visioniastestseries2024
5

● The presence of a strong vertical wind shifting allows nature to replenish the fertility
wedge disturbs the vertical transport of the soil through natural processes; land
of latent heat (essential for cyclone productivity in this type of agriculture is low as
formation, which is basically a heat the farmer does not use fertilisers or other
engine). Hence, Option 4 is incorrect modern inputs. It is known by different names
Reference: NCERT, Class 11th, India Physical in different parts of the world.
Environment, Chapter 6 Pair 1 is correctly matched: ‘Milpa’ in
Mexico and Central America
10. Ans: B Pair 2 is correctly matched: ‘Conuco’ in
Exp: Venzuela
Statement 1 is correct: About 140 million Pair 3 is incorrectly matched: ‘Ray’ in
years ago, the subcontinent was located as far Vietnam
south as 50◦ S latitude. The Tethys Sea Pair 4 is correctly matched: ‘Ladang’ in
separated the Indian Plate and the Eurasian Indonesia
Plate. The Tibetan block was a part of the ‘Roca’ in Brazil
Asiatic landmass. The Indian plate moved ‘Masole’ in Central Africa
northwards and collided with the Eurasian plate
about 40-50 million years ago, causing rapid
uplift of the Himalayas. Therefore, it is a
continent-continent convergence.
Statement 2 is incorrect: The formation of the
Deccan Trap started somewhere around 60
million years ago and continued for a long
period of time. Note that the subcontinent was
still close to the equator. From 40 million
years ago and thereafter, the formation of
the Himalayas took place.
Statement 3 is correct: About 140 million
years before the present, the subcontinent was
located as far south as 50°S latitude. During the Reference: NCERT, Class 10th, Contemporary
movement of the Indian plate towards the India-II, Chapter 4

E
Asiatic plate, a major event that occurred was

IN
the outpouring of lava and the formation of the 12. Ans: D
Deccan Traps (60 million years ago), at that Exp:
time the Indian plate was situated close to the The present atmosphere of our planet Earth has
L
equator. evolved through the following successive
N

Reference: NCERT, Class 11th, Fundamentals stages:


of Physical Geography, Chapter 4 First stage: The loss of the primordial
.O

atmosphere through the impact of the solar


11. Ans: C wind is a significant stage in the evolution of
Exp: the present Earth's atmosphere. Primordial
L

Primitive subsistence agriculture is practised on Earth’s atmosphere consisted mainly of


IA

small patches of land with the help of primitive hydrogen and helium; it was lost into outer
tools like hoe, dao and digging sticks, and space due to the impact of solar winds.
family/ community labour. This type of farming Second stage: Subsequently, the gases and
R

depends upon rainfall, the natural fertility of the water vapour that contributed to the reformation
E

soil and the suitability of other environmental of Earth's early atmosphere were released from
conditions for the crops grown. It is a ‘slash the interior of the solid Earth through a
T

and burn’ agriculture. Farmers clear a patch process called degassing.


A

of land and produce cereals and other food Third stage: The emergence of photosynthetic
crops to sustain their families. When the soil bacteria around 3000-2500 million years ago
M

fertility decreases, the farmers shift and clear a marked an important turning point in the
fresh patch of land for cultivation. This type of evolution of Earth’s atmosphere, as these
C
S
P

https://t.me/visioniastestseries2024
6

organisms used sunlight, carbon dioxide, and Russia, Canada, China, the USA, Brazil, and
water to produce oxygen through Australia occupy a larger area than India. In
Photosynthesis. This led to a gradual increase comparison, the 8th largest country is
in oxygen levels, first in Earth’s oceans. Argentina.
Finally, when the primordial oceans were Reference: NCERT, Class 11th, India Physical
saturated with oxygen around 2000 million Environment, Chapter 1
years ago, the eventual release of oxygen into
the atmosphere gradually led to the present 15. Ans: B
conditions. Exp:
Reference: NCERT, Class 11th, Fundamentals Statement 1 is incorrect: Endogenic
of Physical Geography, Chapter 2 geomorphic processes are geological
processes that originate from within the Earth
13. Ans: A and shape the Earth's surface through various
Exp: internal mechanisms. They are mainly land-
Statement-I is correct: With the increase in building forces originating from the interior of
distance from the sea, the force of the cyclone the earth.
decreases. In India, the force of the cyclone Statement 2 is correct: Due to variation in
decreases with an increase in distance from the geothermal gradients and heat flow from
Bay of Bengal and the Arabian Sea. So, the within, crustal thickness and strength, the
coastal areas are often struck by severe cyclonic action of endogenic forces is not uniform,
storms with an average velocity of 180 km/h, and hence the tectonically controlled original
and this speed decreases landwards. crustal surface is uneven.
Statement-II is correct: A tropical cyclone or Reference: NCERT, Class 11th, Fundamentals
hurricane is like a heat engine that is energised of Physical Geography, Chapter 5
by the release of latent heat on condensation of
moisture (gathered by air while moving over 16. Ans: A
the oceans and seas), and this is one of the Exp:
reasons why a tropical cyclone dissipates on Statement 1 is correct: In agriculture, water is
reaching the land as this vast and continuous mainly used for irrigation. Irrigation is needed
supply of moisture is cut off. because of spatio-temporal variability in
Hence, Both Statement-I and Statement-II rainfall in the country. The large tracts of the

E
are correct and Statement-II is the correct country are deficient in rainfall and are drought-

IN
explanation for Statement-I prone. North-western India and the Deccan
Reference: NCERT, Class 11th, India Physical plateau constitute such areas. The water need of
Environment, Chapter 6 certain crops also makes irrigation necessary.
L
For instance, the water requirement of rice,
N

14. Ans: B sugarcane, jute, etc. is very high which


Exp: means they are water-intensive and their
.O

Statement 1 is incorrect: India's location lies need can be met only through irrigation.
entirely in the northern hemisphere, where the Statement 2 is incorrect: In Punjab, Haryana
mainland extends between latitudes 8° 4' and and western Uttar Pradesh, more than 85 per
L

37° 6' north, longitudes 68° 7' and 97° 25' east. cent of their net sown area is under
IA

When measured in kilometres, it is about 3,214 irrigation. Wheat and rice are grown mainly
km from north to south between the extreme with the help of irrigation in these states. Of the
latitudes and about 2,933 km from east to total net irrigated area 76.1 per cent in Punjab
R

west between the extreme longitudes. and 51.3 per cent in Haryana are irrigated
E

Statement 2 is incorrect: When compared in through wells and tubewells.


degrees, the longitudinal and latitudinal extent Reference: NCERT, Class 12th, India People
T

of India are roughly about 30 degrees. Hence, and Economy, Chapter 4


A

the latitudinal extent doesn't far exceed the


longitudinal extent.
M

Statement 3 is correct: India, in terms of


area, is the 7th largest country in the world.
C
S
P

https://t.me/visioniastestseries2024
7

17. Ans: C 19. Ans: C


Exp: Exp:
Pair 1 is correct: Nebular Hypothesis - This Statement 1 is correct: Meteorological
hypothesis was first proposed by Immanuel drought is a situation when there is a prolonged
Kant. The nebular hypothesis is a widely period of inadequate rainfall marked with mal-
accepted scientific explanation for the distribution of the same over time and space.
formation of our solar system. It proposes that Statement 2 is correct: Agricultural drought is
our solar system originated from a giant cloud also known as soil moisture drought,
of gas and dust called a nebula, and planets characterised by low soil moisture that is
were formed out of gaseous matter associated necessary to support the crops, thereby
with the Sun in the early stages. resulting in crop failures.
Pair 2 is correct: Big Bang theory- Edwin Statement 3 is correct: Hydrological drought
Hubble gives evidence about the expanding results when the availability of water in
universe. The Big Bang theory is the different storages and reservoirs like aquifers,
prevailing scientific explanation for the origin lakes, reservoirs, etc., falls below what the
and evolution of the universe. It suggests that precipitation can replenish.
the universe began as a singularity—an Statement 4 is incorrect: Ecological drought
infinitely hot and dense point that exploded occurs when the productivity of a natural
violently, resulting in the formation of the ecosystem fails due to a shortage of water, and
expanding Universe. as a consequence of this ecological distress, the
Pair 3 is correct: Binary Theory- This theory, ecosystem is damaged.
also called the Planetesimal hypothesis, was Reference: NCERT, Class 11th, India Physical
given by Chamberlain and Moulton in 1900. Environment, Chapter 6
According to this theory, a star collided with
the Sun. As a result, some matter separated 20. Ans: B
from the Sun. After the star moved away, the Exp:
separated material gradually formed the Endogenic geomorphic processes are
planets. geological processes that originate within the
Reference: NCERT, Class 11, Fundamentals Earth and shape the Earth's surface through
of Physical Geography, Chapter II various internal mechanisms. These processes
are driven by forces and energy sources that

E
18. Ans: A originate within the Earth. They include:

IN
Exp: Diastrophism- all processes that move, elevate
Statement-I is correct: The sun rises around or build up portions of the earth’s crust,
2 hours earlier in Dibrugarh and other including:
L
North-East States as compared to Jaisalmer (i) Orogenic processes involving mountain
N

or the westernmost states of Gujarat and building through severe folding


Rajasthan. (ii) Epeirogenic processes involving uplift or
.O

Statement-II is correct: Longitudinal extent of warping of large parts of the earth’s crust
India is nearly 30 degrees between the (iii) Earthquakes, involving local relatively
easternmost and westernmost extremes. This minor movements
L

results in a time difference of nearly two hours (iv) Plate tectonics involving horizontal
IA

between the easternmost and the westernmost movements of crustal plates.


parts of our country. Volcanism: includes the movement of molten
The time along the Standard Meridian of India rock (magma) onto or toward the earth’s
R

(82°30'E) passing through Mirzapur (in Uttar surface, resulting in the formation of many
E

Pradesh) is taken as the standard time for the intrusive and extrusive landforms.
whole country. Weathering and Karst processes are types of
T

Hence, both Statement-I and Statement-II exogenic geomorphic processes.


A

are correct, and Statement-II is the correct Reference: NCERT, Class 11th, Fundamentals
explanation for Statement-I. of Physical Geography, Chapter 5
M

Reference: NCERT, Class 11th, India Physical


Environment, Chapter 1
C
S
P

https://t.me/visioniastestseries2024
8

21. Ans: B Hence, both Statement-I and Statement-II


Exp: are correct, and Statement-II is the correct
India has three cropping seasons — rabi, kharif explanation for Statement-I.
and zaid. Zaid season is a short season during Reference: NCERT, Class 6th, The Earth- Our
the summer months in between the rabi and the Habitat, Chapter 1
kharif seasons. It begins after the harvesting of
rabi crops. Options 1 and 3 are correct: The 23. Ans: D
cultivation of watermelons, cucumbers, Exp:
vegetables and fodder crops during this Statement 1 is incorrect: Hazard can be
season is done on irrigated lands. However, this defined as a dangerous condition or event, that
type of distinction in the cropping season does is a threat or has the potential for causing injury
not exist in southern parts of the country. There, to life or damage to property or the
the temperature is high enough to grow tropical environment.” Hazards can be grouped into two
crops during any period of the year, provided broad categories, namely natural and
the soil moisture is available. Therefore, in this manmade.
region, the same crops can be grown thrice in Statement 2 is incorrect: Vulnerability can be
an agricultural year, provided there is sufficient defined as, the extent to which a community,
soil moisture. structure, service or geographic area is likely to
Options 2 and 4 are incorrect: Kharif crops be damaged or disrupted by the impact of a
are grown with the onset of monsoon in particular hazard, on account of their nature,
different parts of the country, and these are construction, and proximity to hazardous
harvested in September-October. Important terrains or a disaster-prone area. Vulnerability
crops grown during this season are paddy, is broadly categorized into physical and socio-
maize, jowar, bajra, tur (arhar), moong, urad, economic vulnerability.
cotton, jute, groundnut and soyabean. Reference: NCERT, Class 11th, India Physical
Reference: NCERT, Class 12th, India: People Environment, Chapter 6
and Economy, Chapter 3
24. Ans: B
22. Ans: A Exp:
Exp: The air containing moisture to its full capacity
Statement-I is correct: The terrestrial at a given temperature is said to be saturated,

E
planets in our solar system include Mercury, i.e., the relative humidity is 100%. The

IN
Venus, Earth, and Mars. The terrestrial temperature at which saturation (relative
planets, also known as the inner planets or humidity of 100%) occurs in a given sample of
rocky planets, are smaller than the Jovian air is known as the dew point. Condensation
L
planets, also known as the outer planets or gas takes place when the dew point is lower than the
N

giants. freezing point as well as higher than the


Statement-II is correct: Terrestrial planets freezing point. Dew is formed when the dew
.O

formed from metal and rock-based point is above the freezing point.
planetesimals, which were too small to hold Reference: NCERT, Class 11th, Fundamentals
onto much of the solar nebula's rich hydrogen of Physical Geography, Chapter 10
L

and helium gas, thus remaining smaller in size.


IA

The Jovian planets, on the other hand, 25. Ans: D


originated beyond the frost line far from the Exp:
sun, where there was abundant ice and rocks. Tsunamis are giant waves caused by
R

Their cores quickly accumulated sizable ice and earthquakes or volcanic eruptions under the sea.
E

rock clumps, growing to massive proportions, Out in the depths of the ocean, tsunami waves
eventually becoming so large that their do not dramatically increase in height. But as
T

powerful gravity allowed them to pull in a lot the waves travel inland, they build up to higher
A

of abundant hydrogen and other gases. Hence, and higher heights as the depth of the ocean
they are larger than terrestrial planets and are decreases. These waves which often affect
M

also known as the Gas Giants. distant shores are caused by:
C
S
P

https://t.me/visioniastestseries2024
9

● Rapid displacement of water from the the impact of cyclones, we need to take the
lake or the sea either by seismic activity following measures:
or Fault movements on the sea floor. ● Coastal belt plantation: Green belt
Hence, Option 1 is correct. plantation along the coastal line in a
● Landslides. Hence, Option 2 is scientific interweaving pattern can
correct. reduce the impact of a cyclone. Forests
● Volcanic eruptions. Hence, Option 3 is act as a buffer zone against strong
correct. winds and Storm surges. Hence,
● Large meteoroid impacts. Hence, Option 1 is correct
Option 4 is correct. ● Hazard mapping: Cyclones can be
predicted several days in advance. Past
records and paths can give the pattern
of occurrence, and a hazard map
prepared accordingly is useful in
estimating the severity of the cyclone
and in planning mitigation strategies.
Hence, Option 2 is correct
● Land use control: Land use control is
designed so that the least critical
activities are placed in vulnerable
areas. Hence, Option 3 is correct
Reference: NCERT, Class 11th, India Physical
● Engineered structures: Sturdy
Environment, Chapter 6
structures with built-in fail-safes can
help in disaster risk reduction in
26. Ans: B
cyclone prone areas. Hence, Option 4
Exp:
is correct
Statement 1 is incorrect: The atmosphere
Reference: NCERT, Class 11th, India Physical
consists of different layers with varying
Environment, Chapter 6
densities and temperatures. Density is highest
near the surface of the earth and decreases with
28. Ans: D
increasing altitude.
Exp:

E
Statement 2 is correct: The atmosphere has a
Condensation is the process of changing water
sufficient capacity to keep small solid particles,

IN
vapour into tiny droplets of water or ice
which may originate from different sources and
crystals. In our atmosphere takes place in two
include sea salts, fine soil, smoke-soot, ash,
situations, firstly, when the temperature is
pollen, dust and disintegrated particles of
L
below freezing point or 0°C, and secondly,
meteors. A higher concentration of dust
N

when it is above freezing point, so the outcome


particles is found in subtropical and temperate
of the condensation can be classified into two
regions due to dry winds compared to
.O

broad groups as given below:


equatorial and polar regions.
(i) White frost, snow, and some clouds are
Reference: NCERT, Class 11th, Fundamentals
formed when the temperature is below the
L

of Physical Geography, Chapter 7


freezing point.
IA

(ii) Dew, mist, fog, smog, and some clouds are


27. Ans: D
formed when the temperature is above the
Exp:
freezing point.
R

Cyclone is a region of low atmospheric pressure


Reference: NCERT, Class 11th, Fundamentals
surrounded by high atmospheric pressure
E

of Physical Geography, Chapter 10


resulting in swirling atmospheric disturbance
T

accompanied by powerful winds blowing in


29. Ans: D
anticlockwise direction in the Northern
A

Exp:
Hemisphere and in the clockwise direction in
Cyclones occur mainly in the tropical and
the Southern Hemisphere.
M

temperate regions of the world. Cyclones are


They cause a lot of Damage to the lives and
property of human beings. Therefore, to reduce
C
S
P

https://t.me/visioniastestseries2024
10

called by various names in different parts of the of Rajasthan has been an important factor
world such as: in the growth of the rabi crops.
● Hurricanes- North Atlantic Ocean, the Reference: NCERT, Class 10th,
Eastern North Pacific Ocean, and the Contemporary India-II, Chapter 4
Central North Pacific Ocean. Hence,
Pair 1 is incorrectly matched. 32. Ans: D
● Typhoons - China and Japan Hence, Exp:
Pair 2 is incorrectly matched The singularity represents one with an
● Tropical cyclones - Indian Ocean. unimaginably small volume, infinite
● Willie-Willie- Australia. Hence, Pair temperature, and density. As the singularity
3 is incorrectly matched. exploded violently and rapidly expanded, it
● Tornado- North and South America. resulted in the formation of the universe as we
Hence, Pair 4 is incorrectly matched. know it today. The expansion and cooling of the
Reference: NCERT, Class 11th, India Physical universe allowed for the formation of matter,
Environment, Chapter 6 the emergence of fundamental forces, the
creation of atoms, and eventually, the formation
30. Ans: B of galaxies, stars, and planets.
Exp: Reference: NCERT, Class 11th, Fundamentals
Diastrophism: large-scale deformation of of Physical Geography, Chapter 1
Earth’s crust by natural processes, which leads
to the formation of continents and ocean 33. Ans: C
basins, mountain systems, plateaus, rift Exp:
valleys, and other features by mechanisms Statement 1 is incorrect: The state of Manipur
such as lithospheric plate movement i.e. plate is bordered by Nagaland to the North,
tectonics, orogenesis, epeirogenesis etc. Mizoram to the South and Assam to the West.
Orogenesis involves mountain building through It shares its international borders with
severe folding and faulting, and the Epeirogenic Myanmar, which lies towards its east.
process involves the uplift or warping of a large Manipur doesn't share its borders with Tripura.
part of the earth's crust. Statement 2 is incorrect: Among the North
Reference: NCERT, Class 11th, Fundamentals East states, the tropic of cancer passes only
of Physical Geography, Chapter 5 through the states of Tripura and Mizoram.

E
The capital of Manipur - Imphal, is located far

IN
31. Ans: B away from it.
Exp:
Statement 1 is incorrect: Rabi crops are
L
sown in winter from October to
N

December and harvested in summer


from April to June. Some of the important
.O

rabi crops are wheat, barley, peas, gram and


mustard. Though these crops are grown in
large parts of India, states from the north
L

and north-western parts such as Punjab,


IA

Haryana, Himachal Pradesh, Jammu and


Kashmir, Uttarakhand and Uttar Pradesh
are important for the production of wheat
R

and other rabi crops.


E

Statement 2 is correct: The availability


of precipitation during winter months
T

due to the western temperate cyclones


A

helps in the success of rabi crops. The Fig: Manipur and Tropic of Cancer
success of the green revolution in Punjab, Statement 3 is incorrect: The state of Assam
M

Haryana, western Uttar Pradesh and parts shares its borders with the maximum
number of states in the North East. Manipur
C
S
P

https://t.me/visioniastestseries2024
11

shares its borders with only the states of Assam, Pair 4 is incorrectly matched: The state of
Nagaland and Mizoram. Uttar Pradesh doesn't share its border with
Reference: NCERT, Class 11th, India Physical China. The states sharing their borders with
Environment, Chapter 1 China are Uttarakhand, Himachal Pradesh,
Arunachal Pradesh, and Sikkim.
34. Ans: C
Exp:
Statement 1 is incorrect: Orographic or
Relief rain occurs when a saturated air mass
is lifted across a topographic barrier, e.g., a
mountain.
Statement 2 is incorrect: The air mass is
forced to rise and expand as air mass rises
against the windward slope. As a result, the
temperature falls below the dew point and the
moisture condenses to form clouds, causing
widespread rain on the windward slopes of the
mountain range. However, when these winds
Map: India and the neighbouring countries.
descend along the leeward slopes, they get
Reference: NCERT, Class 11th, India Physical
warm and cause very little rain. The region
Environment, Chapter 1
lying on the leeward side of the mountain
receives little rainfall and is called the
36. Ans: A
rainshadow area.
Exp:
Convergence or Cyclonic Rainfall is
Statement 1 is incorrect: The troposphere is
associated with warm and cold fronts (not
the lowermost layer of the atmosphere. Its
Orographic/Relief) Rain.
average height is 13 km and extends roughly to
a height of 8 km near the poles and about 18 km
at the equator. The thickness of the
troposphere is greatest at the equator
because heat is transported to great heights by

E
strong convectional currents. This layer
contains dust particles and water vapour.

IN
Statement 2 is incorrect: Changes in climate
and weather take place in the troposphere. This
L
Figure: Diagram of Orographic/ Relief includes changes in atmospheric concentration
Rainfall of various gases, temperature and humidity, etc.
N

Reference: NCERT, Class 11th, Fundamentals Statement 3 is correct: The temperature in this
of Physical Geography, Chapter 10 layer decreases at a rate of 1 degree Celsius
.O

every 165 metres of height. This is called the


35. Ans: D normal lapse rate.
L

Exp: Reference: NCERT, Class 11th, Fundamentals


Pair 1 is correctly matched: The states of Physical Geography, Chapter 7
IA

sharing borders with Nepal are Uttar Pradesh,


Uttarakhand, Sikkim, West Bengal and Bihar. 37. Ans: B
R

Pair 2 is incorrectly matched: The states of Exp:


Assam, Meghalaya, Tripura, West Bengal, and Flood happens when land that is normally dry
E

Mizoram share their borders with Bangladesh. gets submerged. There are several causes of
T

Manipur doesn't share its borders with floods and they differ from region to region.
Bangladesh. Some of the major causes are:
A

Pair 3 is correctly matched: Bhutan shares its ● Heavy rainfall


borders with 4 states of India i.e. Arunachal ● Heavy siltation of the river bed reduces
M

Pradesh, Sikkim, Assam and West Bengal. the water carrying capacity of the river
channels and can lead to floods. We
C
S
P

https://t.me/visioniastestseries2024
12

desilt river beds, as a flood mitigation


measure. Hence, Option 1 is
incorrect.
● Landslides can cause floods by
blocking the flow of a stream or river.
Hence, Option 2 is correct.
● In areas prone to cyclones, strong
winds accompanied by heavy
downpours cause storm surges that lead
to floods.
● Earthquakes can cause floods by river
avulsions and river course shifting.
Hence, Option 3 is correct.
Reference: NCERT, Class 11th, India Physical
Environment, Chapter 6 (Fig: Indian Standard Meridian)
Reference: NCERT, Class 11th, India Physical
38. Ans. D Environment, Chapter 1
Exp.
Tropical Savannah region's typical climatic 40. Ans: A
feature is a long dry and short wet season. Exp:
Temperature is high throughout the year and Denudation is a broad term that encompasses
the diurnal range of temperature is maximum in all exogenic processes (weathering and
the dry season. The average temperature ranges erosion). Its rate is determined by the physical
from 25 to 30 degrees Celsius. (folds, faults, orientation and inclination of
beds, presence or absence of joints, bedding
planes, hardness or softness of constituent
minerals, permeability) and chemical properties
of the rocks.

E
IN
L
Table: Koeppen climate Classification.
N

Reference: NCERT, Class 11th, Fundamentals Fig: Denudational Processes and the main
driving force behind them.
.O

of Physical Geography, Chapter 11


Reference: NCERT, Class 11th, Fundamentals
39. Ans: A of Physical Geography, Chapter 5
L

Exp:
41. Ans: A
IA

The longitude of 82° 30°E is considered the


Standard Meridian of India. The time at the Exp:
Standard Meridian is regarded as the time for Rice is a staple food for the overwhelming
R

the whole country and is called the Indian majority of the population in India. Though it is
Standard Time or IST. considered to be a crop of tropical humid areas,
E

The cities of Lucknow, Raipur and it has about 3,000 varieties, which are grown in
different agro-climatic regions. These are
T

Hyderabad lie on the western side of the


Standard Meridian of India. Some major successfully grown from sea level to about
A

state capitals to the east of the IST meridian 2,000 m altitude and from humid areas in
are Patna, Kolkata, Bhubaneswar, Ranchi etc. eastern India to dry but irrigated areas of
M

Punjab, Haryana, western U.P. and northern


Rajasthan. In southern states and West Bengal,
C
S
P

https://t.me/visioniastestseries2024
13

the climatic conditions allow the cultivation of Pair 2 is correctly matched: Mid-Oceanic
two or three crops of rice in an agricultural year. Ridge Volcanoes are a part of the Mid-Oceanic
Statement 1 is correct: About one-fourth of Ridge System and are formed in oceanic areas.
the total cropped area in the country is under The central portion of this ridge experiences
rice cultivation. West Bengal, Uttar Pradesh, frequent eruptions as a result of Seafloor
and Punjab are the leading rice-producing states Spreading. As the plates separate, the molten
in the country. The yield level of rice is high in rock rises to the seafloor, producing enormous
Punjab, Tamil Nadu, Haryana, Andhra Pradesh, volcanic eruptions of basalt.
Telangana, West Bengal and Kerala. In the first Pair 3 is correctly matched: Shield volcanoes
four of these states, almost the entire land under are mostly made up of basalt, a type of lava that
rice cultivation is irrigated. has a lower viscosity and it is because of this
Statement 2 is incorrect: Punjab and reason that these volcanoes are not steep. They
Haryana are not traditional rice-growing become explosive if water somehow gets into
areas. Rice cultivation in the irrigated areas of the vent; otherwise, they are characterised by
Punjab and Haryana was introduced in the low explosivity. The upwelling lava moves in
1970s following the Green Revolution. the form of a fountain, throws out the cone
Genetically improved varieties of seed, at the top of the vent, and develops into a
relatively high usage of fertilisers and Shield Cone. Ex- Hawaiian volcano is an
pesticides and lower levels of susceptibility of example of a shield volcano.
the crop to pests due to dry climatic conditions Pair 4 is correctly matched: Dormant
are responsible for higher yield of rice in this Volcanoes are those that have not erupted in a
region. The yield of this crop is meager in very long time. Dormancy occurs because
rainfed areas of Madhya Pradesh, Chhattisgarh magma from the Earth's mantle can no longer
and Odisha. reach the volcano due to the continuous shifting
of the tectonic plates.
Reference: NCERT, Class 11th, Fundamentals
of Physical Geography, Chapter 3

43. Ans: D
Exp:
Statement 1 is correct: India's location lies

E
entirely in the northern hemisphere, where the

IN
extension of the mainland is between latitudes
8° 4' and 37° 6' North, longitudes 68° 7' and 97°
25' East. Looking at the latitudinal extent, the
L
southern part of the country is situated
N

within the tropics, and the northern half is


situated in the sub-tropical zone or the warm
.O

temperate zone.
Map: Rice growing areas of India. Statement 2 is correct: India has a vast
Reference: NCERT, Class 12th, India: People latitudinal extent which ranges more than 3000
L

and Economy, Chapter 3 km, from Kashmir to Kanyakumari. As a result,


IA

there are variations in natural vegetation,


42. Ans: D climate, landforms etc., across India.
Exp: Reference: NCERT, Class 11th, India Physical
R

Pair 1 is correctly matched: Composite Environment, Chapter 1


E

Volcanoes are characterised by the eruption of


cooler and viscous lavas, often resulting in 44. Ans: A
T

explosions. These explosive eruptions lead to Exp:


the accumulation of large quantities of
A

Statement 1 is correct: The fog is a cloud with


pyroclastic material and ashes near the vent its base at or very near the ground in which
M

openings. These continuous deposits of condensation takes place around nuclei


pyroclastic materials lead to the formation of provided by the dust, smoke, and salt particles.
C

layers, giving the volcanoes a mountain shape.


S
P

https://t.me/visioniastestseries2024
14

They are prevalent where warm currents of 46. Ans: D


air come into contact with cold currents. Exp:
Statement 2 is incorrect: One of the Statement-I is incorrect: The solar output
differences between mist and fog is that mist received by the top of the earth’s atmosphere is
contains more moisture than fog. Mists are maximum on the 3rd of January. Because, on
frequent over mountains as the rising warm air the 3rd of January, the earth is nearest to the sun
up the slopes meets a cold surface. so it receives maximum insolation.
Reference: NCERT, Class 11th, Fundamentals Statement-II is correct: The solar output
of Physical Geography, Chapter 10 received at the top of the atmosphere varies
slightly in a year due to the variations in the
45. Ans: B distance between the Earth and the sun. During
Exp: its revolution around the Sun, the Earth is
To move from Imphal (Manipur) to the city of farthest from the Sun on the 4th of July. This
Shimla (Himachal Pradesh), one has to move position of the earth is called aphelion. On 3rd
through a minimum of 6 states, including origin January, the earth is the nearest to the Sun. This
and destination States, namely: position is called perihelion. As the insolation
1. Manipur (origin State) received is inversely proportional to the
2. Assam distance from the Sun, the annual insolation
3. West Bengal received by the earth on 3rd January is slightly
4. Bihar more than the amount received on 4th July
5. Uttar Pradesh Reference: NCERT, Class 11th, Fundamentals
6. Himachal Pradesh (destination of Physical Geography, Chapter 8
State)
The state of Himachal Pradesh shares its 47. Ans: B
borders with the union territories of Ladakh and Exp:
Jammu and Kashmir to its north and with The northern-most range of the Himalayas is
Punjab, Haryana, and Uttarakhand. Himachal known as the ‘Great Himalayas’ or ‘the
Pradesh also shares a very narrow border Himadri’. It contains all prominent Himalayan
with the state of Uttar Pradesh between Behat peaks. Some of the important Himalayan peaks
tehsil, Saharanpur district (UP)and Paonta and their location are:
Sahib tehsil, Sirmaur district (HP). Mt. Everest - Nepal

E
Kanchenjunga - India

IN
Point 1 is incorrect: Makalu - Nepal
Point 2 is incorrect: Dhaulagiri - Nepal
Point 3 is correct: Nanga Parbat - India
L
Point 4 is incorrect: Annapurna - Nepal
N

Nanda Devi - India


Point 5 is correct: Kamet - India
.O
L
IA
R
E
T
A

Map: India political


Reference: NCERT, Class 11th, India Physical
M

Environment, Chapter 1
C
S
P

https://t.me/visioniastestseries2024
15

originates from a glacier close to Bokhar Chu


(31°15' N latitude and 81°40' E longitude) in the
Tibetan region. It is called "Singi Khamban,"
or the Lion's Mouth, in Tibet.
It flows between the Ladakh and Zaskar peaks
in the northwest direction before passing
through Baltistan and Ladakh. It slashes
through the Ladakh range, creating a stunning
gorge close to Gilgit in Jammu and Kashmir. It
enters Pakistan near Chilas, in the Dardistan
Maps: Important Himalayan Peaks. region. The river flows southward and receives
Reference: NCERT, Class 9th, ‘Panjnad’ a little above Mithankot. The
Contemporary India, Chapter 2 Panjnad is the name given to the five rivers of
Punjab, namely the Satluj, the Beas, the Ravi,
48. Ans: D
the Chenab, and the Jhelum.
Exp:
Some of the factors that influence the
movement of ocean water-
● Temperature- The heating by solar
energy causes the water to expand. Warm
water from the equator slowly moves
along the surface towards the poles, while
cold water from the poles slowly creeps
along the bottom of the sea towards the
equator.
● Gravity tends to pull the water down the
pile, creating gradient variations.
● The earth’s rotation and Coriolis force-
The Coriolis force intervenes and causes
the water to move to the right in the

E
northern hemisphere and to the left in the
southern hemisphere.

IN
● Wind blowing on the surface of the ocean
pushes the water to move. Reference: NCERT, Class 11th, India Physical
L
● The obstruction due to landmass- Environment, Chapter 3
hinders the direction of flow of the ocean
N

current and divides the ocean current, 50. Ans: C


.O

which in turn flows in a different direction. Exp:


Example: The south equatorial current in Statement 1 is incorrect: Well-integrated flow
the Atlantic Ocean is obstructed by the with numerous tributaries and well-developed
L

South American continent, and the South floodplains are the features of the old stage of
IA

equatorial current divides to create the the running water regimes.


Brazilian current, which flows in the south Statement 2 is correct: Waterfalls and rapids
Atlantic Ocean. may exist wherever rivers encounter hard rocks
R

Reference: NCERT, Class 11th, Fundamentals in their youth.


of Physical Geography, Chapter 13
E

Statement 3 is correct: The Himalayan


region's young rivers are often dynamic and
T

49. Ans: B play a significant role in shaping the


Exp:
A

landscape. They erode and transport vast


Indus river has a total length of 2,880 km (in amounts of sediment, creating impressive
M

India, 1,114 km) and drains an area of gorges, canyons, and valleys as they flow
11,65,000 sq km (in India 321,289 sq km). It through the rugged terrain.
C
S
P

https://t.me/visioniastestseries2024
16

Reference: NCERT, Class 11th, Fundamentals Statement 1 is incorrect: The famous Kashmir
of Physical Geography, Chapter 5 valley is located between the Great
Himalayas and the Pir Panjal range. It also
51. Ans: C has the renowned Dal Lake.
Exp: Statement 2 is correct: Kashmir Himalayas
Statement-I is correct: Tertiary activities are are famous for Karewa formations. They are the
related to the service sector. Manpower is an thick deposits of glacial clay and other
important component of the service sector, as materials embedded with moraines. These
most of the tertiary activities are performed by Karewa formations are useful for the
skilled labour, professionally trained experts cultivation of Zafran.
and consultants. Reference: NCERT, Class 11th, Indian
Statement-II is incorrect: The main difference Physical Environment, Chapter 2
between secondary activities and tertiary
activities is that the expertise provided by 54. Ans: B
services relies more heavily on the Exp:
specialised skills, experience and knowledge An oasis is a fertile area located within a
of the workers rather than on the production desert, where subterranean water emerges
techniques, machinery and factory processes. to the surface, creating a lush and habitable
Reference: NCERT, Class 12th, Fundamentals environment. Oases are essentially havens of
of Human Geography, Chapter 6 vegetation and life amidst the harsh, arid
conditions of deserts.
52. Ans: B Oasis are formed by the presence of
Exp: underground aquifers or springs that provide a
Statement-I is correct: A Caldera’s consistent source of water. This water emerges
explosiveness depends on the composition of to the surface due to geological factors, often in
the magma and how readily gas can escape low-lying areas, allowing for the growth of
from it. Higher explosivity also indicates that vegetation and the support of various forms of
the source of the lava (magma chamber) is not life, including agriculture and human
only huge but also in close proximity. settlements.
Statement-II is correct: The eruption of Tafilalet Oasis in Morocco is a large oasis with
cooler and more viscous lava than basalt is a an area of about 13,000 sq. km.

E
characteristic of Composite Cone Volcanoes,

IN
also known as Strato Volcanoes. These
volcanoes have been known to erupt
explosively often. Not only lava but also large
L
quantities of pyroclastic material and ashes find
N

their way to the ground and are deposited near


the vent openings in layers leading to the
.O

formation of mountains which are steeper than


Shield Cone Volcanoes. Examples include Mt.
Rainier, USA; Mt. Krakatoa, Indonesia; Mt.
L

Vesuvius, Italy; Mt. Fuji, Japan; etc.


IA

Both Statement-I and Statement-II are


correct individually but Statement-II is not
Reference: NCERT, Class 7th, Our
the correct explanation for Statement-I
R

Environment Chapter 7
Reference: NCERT, Class 11th, Fundamentals
E

of Physical Geography, Chapter 3


55. Ans: D
T

Exp:
53. Ans: B
Statement 1 is correct: The Ravi is an
A

Exp:
Northwestern Himalayas comprise a series of important tributary of the Indus. It rises west of
M

ranges such as the Karakoram, Ladakh, Zanskar the Rohtang Pass in the Kullu hills of Himachal
and Pir Panjal. Pradesh and flows through the Chamba valley
C
S
P

https://t.me/visioniastestseries2024
17

of the state. Before entering Pakistan and


joining the Chenab near Sarai Sidhu, it drains
the area lying between the southeastern part of
the Pir Panjal and the Dhauladhar ranges.
Statement 2 is correct: The Beas is also an
important tributary of the Indus, originating
from the Beas Kund near the Rohtang Pass at
an elevation of 4,000 m above mean sea level.
The river flows through the Kullu valley, and
forms gorges at Kati and Largi in the
Dhauladhar range. It enters the Punjab plains,
where it meets the Satluj near Harike. Fig: Angle of Sun’s rays with respect to Earth’s
Statement 3 is correct: The Satluj originates in Surface at different latitudes.
the ‘Raksas tal’ near Mansarovar at an altitude Reference: NCERT, Class 11th, Fundamentals
of 4,555 m in Tibet, where it is known as of Physical Geography, Chapter 8
Langchen Khambab. It flows almost parallel
to the Indus for about 400 km before entering 57. Ans: C
India and comes out of a gorge at Rupar. It Exp:
passes through Shipki La on the Himalayan The average annual rainfall in India is about
ranges and enters the Punjab plains. It is an 125 cm, but it has great spatial variations.
antecedent river. It is a very important ● The highest rainfall occurs along the
tributary as it feeds the canal system of the west coast, on the Western Ghats, and
Bhakra Nangal project. in the sub-Himalayan areas in the
Reference: NCERT, Class 11th, India Physical northeast and the hills of Meghalaya,
Environment, Chapter 3 while areas of Rajasthan, the Deccan
Plateau, etc., receive very low rainfall.
56. Ans: D
Hence, Option A is incorrect.
Exp:
Statement-I is incorrect: Insolation received ● The monsoon is the axis around which

E
at Earth’s surfaces decreases as we move revolves the entire agricultural cycle of

IN
towards the poles. India. It is because about 64 percent of
The insolation received at the surface varies the people of India depend on
from about 320 watts/m2 in the tropics to about agriculture for their livelihood, and
L
70 watts/m2 in the poles.
agricultural planning is based on
N

Statement-II is correct: The Sun's rays make


a smaller angle with the Earth's surface as southwest monsoon, but irrigation
.O

latitude increases. The area that the vertical rays also plays an important role in
cover is always less than the area covered by agriculture development of India.
slant rays. The energy is spread as more area is The agricultural prosperity of India
L

covered, resulting in a reduction in the net depends very much on timely and
IA

energy received per unit area. Additionally, adequately distributed rainfall. If it


because the slant rays must penetrate through a
fails, agriculture is adversely affected,
greater depth of the atmosphere, heat losses will
R

be more due to absorption, scattering, and particularly in those regions where


means of irrigation are not developed.
E

diffusion. Hence, with an increase in the


angle of sun rays with respect to the Earth's Hence, Option B is incorrect.
T

surface, insolation increases. ● Winter rainfall by temperate cyclones


A

in north India is highly beneficial for


rabi crops. These are called western
M

disturbances. Hence, Option C is


correct.
C
S
P

https://t.me/visioniastestseries2024
18

● Regional variations in monsoon


climate help in growing various types
of crops. Thus, diversity in crops is
observed in a country like India.
Hence, Option D is incorrect.
Reference: NCERT, Class 11th, India Physical
Environment, Chapter 4

58. Ans: C
Exp:
Pair 1 is incorrect: The Wave frequency is Picture: A Bugyal.
the number of waves passing a given point Reference: NCERT, Class 11th, Indian
during a unit time interval. It is usually Physical Environment
expressed in waves per second.
Pair 2 is incorrect: The highest point of a 60. Ans: A
wave is called the crest, and the lowest point Exp:
of a wave is known as the Trough. Statement 1 is correct: A gorge and a canyon
Pair 3 is incorrect: The wavelength is the are both geological landforms associated with
horizontal distance between two successive erosional features of rivers with deep valleys,
crests. but they have some distinguishing
Wave amplitude is one-half of the wave height characteristics. A gorge is a deep valley with
(not wave frequency). The Wave speed is the very steep to straight sides, and the canyon is
rate at which the wave moves through the characterised by steep step-like side slopes.
water, and is measured in knots (not wave Statement 2 is incorrect: A gorge is almost
length). equal in width at its top and bottom. In
contrast, a canyon is wider at its top than its
bottom.

E
IN
Reference: NCERT, Class 11th, Fundamentals
of Physical Geography, Chapter 13
L
N

59. Ans: D
Exp:
.O

In the native Uttarakhandi dialect, bugyal


refers to a meadow or pasture. These are alpine
L

meadows in the higher reaches. Dayara


Bugyal, located 12000 feet above sea level, is
IA

one of the state's most pristine meadows.


In the Great Himalayan range, the valleys are
R

used by the Bhotias. These are nomadic groups


who migrate to these bugyals during the
E

summer months and return to the valleys during


winter.
T
A

Fig: Gorge (left) and Canyon (right).


Reference: NCERT, Class 11th, Fundamentals
M

of Physical Geography, Chapter 5


C
S
P

https://t.me/visioniastestseries2024
19

61. Ans: D 63. Ans: A


Exp: Exp:
Retail Trading is the business activity The Ganga is the most important river in India,
concerned with the sale of goods directly to the both from the point of view of its basin and its
consumers. Most of the retail trading takes cultural significance. It rises in the Gangotri
place in fixed establishments or stores solely glacier near Gaumukh (3,900 m) in the
devoted to selling. Street peddling, handcarts, Uttarkashi district of Uttarakhand. It is shared
trucks, door-to-door, mail-order, telephone,
by Uttarakhand (110 km) and Uttar Pradesh
automatic vending machines and the
(1,450 km), Bihar (445 km), and West Bengal
internet are examples of non-store retail
trading. (520 km), and falls into the Bay of Bengal.
In its course, it is joined by a large number of
tributaries, both from the right and left banks.

Important left bank tributaries are:


Reference: NCERT, Class 12th, Fundamentals ● Gomti
of human geography, Chapter 6 ● Gandak
● Ghaghra
62. Ans: C
Exp: ● Kosi
Dyke is a type of intrusive landform formed in Important Right bank tributaries are:
● Yamuna

E
a fracture of a pre-existing rock strata. Dykes
can either be Igneous (magmatic) or ● Son

IN
sedimentary in origin. An igneous dyke is ● Damodar
formed when lava flows into a crack and Reference: NCERT, Class 11th, India Physical
solidifies almost perpendicular to the ground, Environment, Chapter 3
L
whereas sedimentary dykes form when
N

sediment fills a pre-existing crack in the rock 64. Ans: C


strata.
.O

Exp:
Spring Tides occur when the sun, the moon,
and the earth are in a straight line. During
L

spring tides, the sun’s tidal force works with the


moon’s tidal force, and the combined pull can
IA

cause the highest and lowest tides. They occur


twice a month, one during the full moon
R

period and another during the new moon


period. Normally, there is a seven-day
E

interval between the spring tides and the


T

neap tides.
A neap tide refers to a period of moderate tides
A

Fig: Intrusive landforms of Magmatic Origin when the sun and moon are at right angles to
Reference: NCERT, Class 11th, Fundamentals each other. It occurs seven days after a spring
M

of Physical Geography, Chapter 3 tide.


C
S
P

https://t.me/visioniastestseries2024
20

Fig: The occurrence of a Spring tide.


Reference: NCERT, Class 11th, Fundamentals
of Physical Geography, Chapter 13

65 Ans: D
Exp:
Statement 1 is incorrect: The Himalayas, Reference: NCERT, Class 11th, Fundamentals
geologically young and structurally fold of Physical Geography, Chapter 8
mountains, stretch over the northern borders of
India. These mountain ranges run in a west-east 67. Ans: C
direction from the Indus to the Brahmaputra. Exp:
Their width varies from about 400 km in
The northern portion of the country experiences
Kashmir to 150 km in Arunachal Pradesh. The
altitudinal variations are greater in the a period of extreme heat and lowering air
eastern half than in the western half. pressure during the summer.
Statement 2 is incorrect: The northern most In July, the ITCZ advances northward,
range of the Himalayas is known as the Great assuming a position centered around 25°N as a
or Inner Himalayas or the Himadri. It is the result of the subcontinent heating up.
most continuous range consisting of the Some of the famous local storms that develop
loftiest peaks with an average height of 6,000 in India in the hot weather season are:
metres. ● Mango Showers: Towards the end of
Reference: NCERT, Class 9th, Contemporary summer, there are pre-monsoon
India, Chapter 2

E
showers, a common phenomenon in
Kerala and coastal areas of Karnataka.

IN
66. Ans: B
Exp: Locally, they are known as mango
Statement-I is correct: The Sun being a high showers since they help in the early
L
temperature body radiates energy mainly in ripening of mangoes. Hence, Pair 1 is
shorter wavelengths, hence the earth’s surface
N

correctly matched.
receives most of its energy in short ● Blossom Shower: With this shower,
.O

wavelengths.
coffee flowers blossom in Kerala and
Statement-II is correct: The energy received
by the earth is known as incoming solar nearby areas. Hence, Pair 2 is
L

radiation, also known as insolation; the incorrectly matched.


● Nor Westers: These are dreaded
IA

amount and the intensity of insolation vary


during the day, in a season and in a year. evening thunderstorms in Bengal and
Hence, Both Statement-I and Statement-II Assam. Their notorious nature can be
R

are correct and Statement-II is not the understood from the local
correct explanation for Statement-I.
E

nomenclature of ‘Kalbaisakhi’, a
T

calamity of the month of Baisakh.


These showers are useful for tea, jute,
A

and rice cultivation. In Assam, these


M

storms are known as Bardoisila.


Hence, Pair 3 is correctly matched.
C
S
P

https://t.me/visioniastestseries2024
21

● Loo: Hot, dry, and oppressing winds characterized by a fixed course, the absence of
blowing in the Northern plains from meanders, and a nonperennial flow of water.
Punjab to Bihar, with higher intensity Statement 3 is correct: Mahi River is the only
between Delhi and Patna. Hence, Pair river in India that cuts the tropic of cancer
4 is correctly matched. twice, once in Madhya Pradesh, where it flows
Reference: NCERT, Class 11th, India Physical west towards Rajasthan, and then enters
Environment, Chapter 4 Gujarat, where it cuts the tropic of cancer for
the second time.
68. Ans: B Reference: NCERT, Class 11th, India Physical
Exp: Environment, Chapter 3
Statement 1 is correct: The Amazon River
Basin lies along the equator, covering 70. Ans: A
multiple South American countries, and is Exp:
renowned for its consistently warm and Statement 1 is correct: Karst topography,
moist climate year-round. Statement 2 is often referred to simply as "karst," is a
incorrect: Maximum daily temperatures in the distinctive landscape that results from the
Amazon Basin hovers around 30 C, and the dissolution of soluble rocks, primarily
daily minimum is generally around 20 C. Thus limestone, gypsum, and dolomite. It has various
the diurnal range of the temperature is around unique surface features, including sinkholes,
10 C and not negligible. caves, disappearing streams, underground
Statement 3 is correct: The Amazon Basin is drainage systems, and limestone pavements.
known for its frequent rainfall, often Any limestone or dolomitic region showing
occurring daily without much advance typical landforms produced by the action of
warning. This region experiences heavy and groundwater through the processes of
sudden rain showers, contributing to its lush solution and deposition is called Karst
rainforest ecosystem. These rain showers are a topography.
key component of the region's unique climate Statement 2 is incorrect: In Karst
and biodiversity. topography, both erosional and depositional
Reference: NCERT, Class 7th, Our factors are active in the formation of
Environment, Chapter 7 landforms like poljes, sinkholes, lapies,

E
limestone pavements, stalactites, stalagmites,

IN
69. Ans: B pillars, etc.
Exp: Reference: NCERT, Class 11th, Fundamentals
Statement 1 is incorrect: The Indian drainage of Physical Geography, Chapter 5 L
system can be classified into two parts:
N
71. Ans: A
● Himalayan drainage system
Exp:
● Peninsular drainage system
.O

The Trans–siberian Railways is a major rail


In this sense, the Peninsular drainage system route of Russia which runs from St. Petersburg
predates the Himalayan drainage system. The in the west to Vladivostok on the Pacific Coast
L

wide, heavily-graded shallow valleys and the in the east passing through Moscow, Ufa,
IA

flat terrain make this clear. Novosibirsk, Irkutsk, Chita and Khabarovsk. It
Statement 2 is correct: Most of the major is the most important route in Asia and the
Peninsular rivers, except Narmada and Tapi, longest double-tracked and electrified
R

flow from west to east. The Chambal, the transcontinental railway in the world. It has
E

Sind, the Betwa, the Ken, and the Son, helped in opening up its Asian region to West
originating in the northern part of the European markets. It runs across the Ural
T

Mountains Ob and Yenisei rivers Chita is an


Peninsula, belong to the Ganga river system.
important agrocentre and Irkutsk is a fur centre.
A

The other major river systems of the Peninsular


There are connecting links to the south. For
drainage are the Mahanadi, the Godavari, the
M

example,
Krishna, and the Kaveri. Peninsular rivers are ● Odessa - Ukraine. Hence, Pair 1 is
C

incorrectly matched.
S
P

https://t.me/visioniastestseries2024
22

● Baku - Azerbaijan. Hence, Pair 2 is 74. Ans: C


incorrectly matched. Exp:
● Ulan Bator - Mongolia. Hence, Pair 3 Ocean currents are streams of water flowing
is incorrectly matched. constantly on the ocean surface in definite
● Tashkent - Uzbekistan. Hence, Pair 4 directions. The ocean currents may be warm or
is correctly matched. cold. Some of the cold currents are as
● Shenyang - Mukden follows- Labrador Current, Falkland
● Beijing - China. Current, Humboldt Current, Oyashio
Reference: NCERT, Class 12th, Fundamentals Current, Canary Current etc.
of Human Geography, Chapter 7 Some of the Warm ocean currents are- The Gulf
Stream, Kuroshio current, Agulhas Current,
72. Ans: B Alaska Current , Florida Current, etc.
Exp:
Statement 1 is incorrect: In the case of S-
waves/shear waves, the particles of the medium
vibrate perpendicular to the wave direction, and
in the case of P-waves/ compressional waves,
particles of the medium vibrate parallel to
the direction of the wave.
Statement 2 is correct: S waves propagate
slower than P waves because P waves are
longitudinal waves and S waves are transverse
waves. P waves apply a force in the direction of Fig: Ocean Currents
propagation. P-waves, because they move Reference: NCERT, Class 11th, Fundamentals
faster, are the first to arrive at the surface. of Physical Geography, Chapter 13
Statement 3 is correct: S waves can travel
only through solid materials, whereas P- 75. Ans: C
waves travel through all the mediums- Exp:
gaseous, liquid, and solid. Statement 1 is correct: One of the factors that
influence the temperature of a place is the
distance of the place with respect to the sea.

E
Compared to land, the sea gets heated slowly
and loses heat slowly i.e. land heats up and

IN
cools down quickly. This phenomenon is due
to differences in the specific heat capacities of
L
land and water.
Statement 2 is correct: The variation in
N

Fig: Propagation of Body Waves


Reference: NCERT, Class 11th, Fundamentals temperature of places near the sea is less
.O

of Physical Geography, Chapter 3 compared to inland places. The places near the
sea come under the moderating influence of the
73. Ans: A sea and land breezes and hence have a smaller
L

Exp: temperature range.


IA

Mizoram is located in the northeastern part Reference: NCERT, Class 11th, Fundamentals
of India. It is bordered by Myanmar in the of Physical Geography, Chapter 8
east and south, Bangladesh in the west and
R

south, and the states of Tripura, Assam in 76. Ans: C


Exp:
E

the northwest and Manipur in the northeast.


Mizoram is also known as the ‘Molassis Statement 1 is correct: A dendritic drainage
T

basin’. It is made up of soft unconsolidated pattern is characterized by a river or stream


system that resembles the branching of tree
A

deposits.
Reference: NCERT, Class 11th, Indian roots. It is one of the most common drainage
M

Physical Environment, Chapter 2 patterns observed in nature. In a dendritic


pattern, smaller tributaries join larger rivers in
C
S
P

https://t.me/visioniastestseries2024
23

a manner similar to the way tree branches and population. Thus, the best fishing grounds in
twigs merge into larger limbs. the world are mainly found in these mixing
Statement 2 is correct: Centripetal rivers zones. For example- the Grand Banks of
flow towards a central point, such as a lake Newfoundland. Moreover, in these mixing
or depression. They are formed in areas zones, the air above the cold current causes the
where the land slopes gently towards the water vapour of the warm current to condense
and form fog. This dense fog reduces
centre. The rivers flow in a circular pattern,
visibility, making navigation challenging.
with the water always flowing towards the
Reference: NCERT, Class 11th, Fundamentals
centre. This type of drainage pattern is common of Physical Geography, Chapter 13
in areas with a depression, such as a volcano
crater or a sinkhole. 79. Ans: A
Exp:
The northern boundary of the Peninsular Block
may be taken as an irregular line running from
Kachchh along the western flank of the Aravali
Range near Delhi and then roughly parallel to
the Yamuna and the Ganga as far as the
Rajmahal Hills and the Ganga delta.
Statement 1 is correct: The Deccan Plateau is
higher in the west and slopes gently eastwards.
The Plateau also has extensions in North
Eastern India, known as the Meghalaya
Plateau, Karbi-Anglong Plateau and North
Cachar Hills. The northeastern parts are
separated by the Malda fault in West Bengal
from the Chotanagpur plateau.
Statement 2 is incorrect: Since the Cambrian
period, the Peninsula has been standing like a
rigid block with the exception of some of its
western coast which is submerged beneath the

E
sea and some other parts which changed due to

IN
tectonic activity without affecting the original
Reference: NCERT, Class 11th, India Physical craton. As a part of the Indo-Australian Plate, it
has been subjected to various vertical
Environment, Chapter 3
L
movements and block faulting. The rift
N

valleys of the Narmada, the Tapi, the


77. Ans: A Mahanadi, and the Satpura block mountains
.O

Exp: are some examples of it.


Epiphytes like Orchids and Bromeliads thrive
in rain forests. Rain forests provide a
L

suitable environment with high humidity,


IA

abundant rainfall, and diverse host plants.


These parasitic plants attach themselves to trees
or other vegetation, obtaining nutrients and
R

water from their hosts.


Reference: NCERT, Class 7th, Our
E

Environment Chapter 7
T

78. Ans: B
A

Exp:
The mixing of warm and cold currents helps
M

Reference: NCERT, Class 11th, Indian


to replenish oxygen and favour the growth of Physical Environment, Chapter 2
plankton, the primary food for the fish
C
S
P

https://t.me/visioniastestseries2024
24

80. Ans: D Reference: NCERT, Class 11th, Fundamentals


Exp: of Physical Geography, Chapter 5
The movement of glaciers is slow, unlike water
flow and results in the formation of typical 81. Ans: D
landforms: Exp:
● Cirques are the most common of Statement 1 is incorrect: The principal modes
landforms in glaciated mountains. The of transportation in the world are land, water,
cirques quite often are found at the air and pipelines. These are used for inter-
heads of glacial valleys. The regional and intra-regional transport, and each
accumulated ice cuts these cirques one (except pipelines) carries both
while moving down the mountain tops. passengers and freight. The significance of a
They are deep, long and wide troughs mode depends on the type of goods and services
or basins with very steep concave to to be transported, the costs of transport and the
vertically dropping high walls at their mode available.
head and sides. A water lake is often Statement 2 is incorrect: Road transport is
seen within the cirques after the glacier cheaper and faster over short distances and
disappears. Such lakes are called cirque for door-to-door services. Railways are most
or tarn lakes. There can be two or more suited for large volumes of bulky materials over
cirques, one leading into another below long distances within a country. High-value,
in a stepped sequence. light and perishable goods are best moved by
● Arete is a narrow, sharp ridge that airways. In a well-managed transport system,
separates two cirques, or glacial these various modes complement each other.
valleys. It forms when two glaciers Reference: NCERT, Class 12th, Fundamentals
erode parallel valleys on opposite sides of Human Geography, Chapter 7
of a mountain or ridge. As the glaciers
erode the rock, they leave behind a 82. Ans: A
sharp crest that is steep and often has Exp:
serrated edges. Statement 1 is incorrect: Only body waves (P
● Truncated Spurs are elongated ridges and S waves) from the shadow zone on the
that extend outward from the sides of a earth’s surface. Surface waves do not form a
glacial valley. They are formed when

E
shadow zone.
the glacier erodes the sides of the Statement 2 is incorrect: The shadow zone of

IN
valley, leaving behind triangular- the S-wave is much larger than that of the P-
shaped landforms that jut out into the waves. The shadow zone of P-waves appears as
valley. a band around the earth between 105° and 145°
L
● Moraines are long ridges of glacial till. away from the epicentre. The shadow zone of
N

Terminal moraines are long ridges of S-waves is not only larger in extent but it is also
debris deposited at the end (toe) of the a little over 40 per cent of the earth's surface.
.O

glaciers. Lateral moraines form along Statement 3 is correct: There are two types of
the sides, parallel to the glacial valleys. body waves - S and P. The shadow zone of S
waves is larger and covers around 40 % of the
L

earth’s surface. The shadow zone of P waves is


IA

smaller (105 - 145 degrees). These shadow


zones overlap with each other between 105 and
145 degrees.
R
E
T
A
M

Fig: Landforms associated with glaciers.


C

Fig: P-wave and S-wave shadow zone


S
P

https://t.me/visioniastestseries2024
25

Reference: NCERT, Class 11th, Fundamentals


of Physical Geography, Chapter 3 85. Ans: D
Exp:
83. Ans: D The air is set in motion due to the differences in
Exp: atmospheric pressure. The air in motion is
Statement 1 is correct: The easterly jet stream called wind. The wind blows from high
sets in along 15°N latitude only after the pressure to low pressure. The wind at the
western jet stream has withdrawn itself from the surface experiences friction from the land. In
region. This easterly jet stream is held addition, the rotation of the earth also affects
the wind's movement. The force exerted due to
responsible for the burst of the monsoon in
the rotation of the earth is known as the
India.
Coriolis force. Thus, the horizontal winds near
Statement 2 is correct: The southwest the earth's surface respond to the combined
monsoon may be seen as a continuation of the effect of three forces – the pressure gradient
southeast trade winds deflected towards the force, the frictional force, and the Coriolis
Indian subcontinent after crossing the Equator. force. In addition, the gravitational force acts
These winds cross the Equator between 40°E downward. Hence, all the factors mentioned
and 60°E longitudes. The shift in the position of above affect the velocity and direction of the
the ITCZ is also related to the phenomenon of wind.
the withdrawal of the westerly jet stream from Reference: NCERT, Class 11th, Fundamentals
its position over the north Indian plain. of Physical Geography, Chapter 9
Reference: NCERT, Class 11th, India Physical
Environment, Chapter 4 86. Ans: C
Exp:
The decomposers are the fungi, bacteria and
84. Ans: D
flagellates, which are especially abundant at the
Exp: bottom of the pond and responsible for breaking
Every three to seven years, a complicated down organic matter and recycling nutrients.
weather system called EI-Nino causes These decomposers play a significant role in
droughts, floods, and other extreme weather nutrient cycling.
conditions in many parts of the world. Fungi: Fungi are known for their role in

E
With the emergence of warm currents off the decomposing complex organic materials, They

IN
coast of Peru in the Eastern Pacific, the system produce enzymes that break down organic
encompasses both oceanic and atmospheric compounds into simpler substances, which can
processes and influences weather in many then be taken up by other organisms, including
L
countries, including India. EI-Nino is plants and algae.
N

essentially an extension of the warm equatorial Bacteria: Bacteria are the most abundant and
diverse group of decomposers in pond
current, which briefly replaces the cold
.O

ecosystems. They play a crucial role in the


Peruvian current or Humboldt current. The
decomposition of organic matter, including the
water temperature along the Peruvian coast is breakdown of dead plants, animal remains, and
L

raised by this stream. organic detritus.


IA

This results in: Flagellates: Flagellates are single-celled


● the distortion of equatorial atmospheric protists with whip-like structures (flagella) that
circulation; they use for movement. Some flagellates are
R

● irregularities in the evaporation of sea heterotrophic and feed on bacteria and organic
E

water particles in the water and sediment. By grazing


on bacteria and other microorganisms,
● reduction in the amount of planktons
T

flagellates help regulate microbial populations


which further reduces the number of
A

and further contribute to nutrient cycling in the


fish in the sea. pond ecosystem.
M

Reference: NCERT, Class 11th, India Physical Reference: NCERT, Class 12th, Biology,
Environment, Chapter 4 Chapter 12
C
S
P

https://t.me/visioniastestseries2024
26

perennial rivers, which have water present


87. Ans: A throughout the entire year; and (b) ephemeral
Exp: rivers, which have seasonal water presence.
The Northern Plains are formed by the alluvial Statement 2 is correct: A catchment is a region
deposits brought by the rivers – the Indus, the of land that gathers water during rainstorms,
Ganga and the Brahmaputra. From the north to often enclosed by hills. The water finds its way
the south, these can be divided into three major into streams and into the ground as it travels
zones: the Bhabar, the Tarai and the alluvial across the terrain, eventually feeding the river.
plains. The alluvial plains can be further In periods of low rainfall, some of this water
divided into the Bhangar and the Khadar. remains underground and gradually feeds the
Bhabar is a narrow belt ranging between 8-10 river.
km parallel to the Shiwalik foothills at the Statement 3 is correct: An area that is drained
break-up of the slope. South of the Bhabar is the by a river and all of its tributaries is known as a
Tarai belt, where most of the streams and river drainage basin. Many diverse watersheds
rivers re-emerge without having any properly make up a river basin.
demarcated channel, thereby creating marshy Statement 4 is correct: There is a drainage
and swampy conditions. The south of Tarai is a basin for each river. A watershed divides two or
belt consisting of old and new alluvial deposits more river basins from one another.
known as the Bhangar and Khadar, Reference: NCERT, Class 11th, India Physical
respectively. Environment, Chapter 3
Reference: NCERT, Class 11th, Indian
Physical Environment, Chapter 2 90. Ans: A
Exp:
88. Ans: B Statement 1 is incorrect: Fjords are a type of
Exp: coastal feature associated with the submergence
Statement 1 is incorrect: The differences in of glacial valleys.
atmospheric pressure produce a force. The rate Statement 2 is incorrect: Lagoon is a shallow
of change of pressure with respect to distance is body of water protected from a larger body of
the pressure gradient. The pressure gradient is water (usually the ocean) by sandbars,
strong where the isobars are close to each associated with low sedimentary coasts.
other and weak where they are apart. Statement 3 is correct: The west coast of

E
Statement 2 is correct: Frictional Force affects India is a high rocky retreating coast

IN
the speed of the wind. It is greatest at the dominated by erosional landforms, whereas the
surface and its influence generally extends up east coast of India is a low sedimentary coast
to an elevation of 1 - 3 km. Over the sea surface, where deposition is dominant.
L
the friction is minimal as compared to the land. Reference: NCERT, Class 11th, Fundamentals
N

Statement 3 is correct: The rotation of the of Physical Geography, Chapter 5


earth about its axis affects the direction of the
.O

wind. This force is called the Coriolis force. 91. Ans: C


The Coriolis force acts perpendicular to the Exp:
pressure gradient force. The pressure gradient Trans–Siberian Railways is one of the major
L

force is perpendicular to an isobar. The higher rail routes of Russia which runs from St.
IA

the pressure gradient force, the more is the Petersburg in the west to Vladivostok on the
velocity of the wind and the larger the Pacific Coast in the east passing through
deflection in the direction of the wind. Moscow, Ufa, Novosibirsk, Irkutsk, Chita and
R

Reference: NCERT, Class 11th, Fundamentals Khabarovsk.


E

of Physical Geography, Chapter 9 Statement 1 is correct: It is the most important


route in Asia and the longest double-tracked
T

89. Ans: C and electrified transcontinental railway in the


A

Exp: world. It has helped in opening up its Asian


Statement 1 is incorrect: According to the region to West European markets.
M

presence of water in river through the year, Statement 2 is correct: It runs across the Ural
rivers are divided into two categories: (a) Mountains, Ob and Yenisei rivers. Chita is an
C
S
P

https://t.me/visioniastestseries2024
27

important agrocentre and Irkutsk is a fur centre 1. Conversion of inorganic material into
on the line. organic material with the help of the
radiant energy of the sun by the autotrophs
2. Consumption of the autotrophs by
heterotrophs
3. Decomposition and mineralisation of
the dead matter to release them back for
reuse by the autotrophs.
Reference: NCERT, Class 12th, Biology,
Chapter 12

94. Ans: D
Map: The Trans Siberian Railway Line. Exp:
Statement 1 is incorrect: The Indian
Reference: NCERT, Class 12th, Fundamentals subcontinent is very big in size and fed by
of Human Geography, Chapter 7 different river systems all around the country.
The Indian drainage system can be divided into
92. Ans: B two categories based on water discharge:
Exp: (i) The Arabian Sea drainage
Post-drift studies provided considerable (ii) The Bay of Bengal drainage.
information that was not available at the time The Delhi Ridge, the Aravalis, and the
Alfred Wegener put forth his concept of Sahyadris separate them from one another.
continental drift. Particularly, the mapping of Majority of the drainage area, which includes
the ocean floor and paleomagnetic studies of the Ganga, Brahmaputra, Mahanadi, Krishna,
rocks from oceanic regions revealed the and other rivers, drains into the Bay of Bengal.
following facts:- The rest of it, which includes the Indus,
Statement 1 is incorrect: All along the mid- Narmada, Tapi, Mahi, and Periyar systems,
oceanic ridges, volcanic eruptions are discharge their water into the Arabian Sea.
common, and they bring vast amounts of lava Statement 2 is incorrect: The majority of
to the surface in this area. rivers that flow across the Peninsular Plateau
originate in the Western Ghats and empty into

E
Statement 2 is incorrect: The oceanic crust
rocks are much younger than the continental the Bay of Bengal, not the Eastern Ghats.

IN
rocks. The age of rocks in the oceanic crust is
nowhere more than 200 million years old. Some L
of the continental rock formations are as old as
3,200 million years.
N

Statement 3 is correct: Recent research has


established that the sediments on the ocean
.O

floor are unexpectedly thin. Before this,


scientists had hypothesised that ocean floors
L

should have a complete sequence of sediments


of much longer duration. However, nowhere
IA

was the sediment column found to be older than


200 million years
R

Reference: NCERT, Class 11th, Fundamentals


of Physical Geography, Chapter 4
E

93. Ans: C
T

Exp:
A

Ecosystems perform several vital functions that


are essential for maintaining ecological
M

balance, and supporting life:


C
S
P

https://t.me/visioniastestseries2024
28

plains cover more than 50% of the Earth’s


Reference: NCERT, Class 11th, India Physical surface.
Environment, Chapter 3 Reference: NCERT, Class 11th, Fundamentals
of Physical Geography, Chapter 4
95. Ans: A
Exp: 97. Ans: D
Statement-I is correct: The pressure Exp:
gradient force is perpendicular to an isobar Secondary productivity is defined as the rate
and the wind blows perpendicular to isobars at of formation of new organic matter by
the equator. consumers.
Statement-II is correct: The Coriolis force Unlike primary productivity, which involves
acts perpendicular to the pressure gradient the synthesis of organic matter by autotrophic
force. The higher the pressure gradient force, organisms like plants and algae through
the higher the velocity of the wind and the photosynthesis, secondary productivity
larger is the deflection in the direction of the involves the production of organic matter by
wind. As a result of these two forces operating consumers like animals that obtain energy by
perpendicular to each other, in the low-pressure consuming other organisms.
areas, the wind blows around it. At the equator, It focuses on the energy transfer from primary
the Coriolis force is zero and the wind blows producers, like plants and algae, to consumers,
perpendicular to the isobars. The low such as herbivores, carnivores, and omnivores.
pressure gets filled instead of getting Reference: NCERT, Class 12th, Biology,
intensified. That is the reason why tropical Chapter 12
cyclones are not formed near the equator.
Hence, Both Statement-I and Statement-II 98. Ans: C
are correct and Statement-II is the correct Exp:
explanation for Statement-I Statement 1 is correct: Water vapour is also a
Reference: NCERT, Class 11th, Fundamentals variable gas in the atmosphere, which decreases
of Physical Geography, Chapter 9 with altitude. In the warm and wet tropics, it
may account for four per cent of the air by
96. Ans: A volume, while in the dry and cold areas of
Exp: deserts and polar regions, it may be less than

E
Option A is correct: A tillite is a sedimentary one percent of the air.

IN
rock formed from deposits or lithification of Statement 2 is correct: Water vapour
glacial till/glaciers. Gondwana-type tillite decreases from the equator towards the poles
sediments are also found in Africa, Falkland because insolation decreases as we move from
L
Island, Madagascar, Antarctica, and Australia. the tropics to the poles, and due to this,
N

These sediments clearly demonstrate that these evaporation also decreases, which results in the
landmasses had remarkably similar histories. decrease of vapour in the atmosphere
.O

The glacial tillite provides unambiguous Statement 3 is correct: It also absorbs parts of
evidence of palaeoclimates and also of the the insolation from the sun and preserves the
drifting of continents. earth’s radiated heat. It, thus, acts like a blanket
L

Option B is incorrect: The submarine trenches allowing the earth neither to become too cold
IA

are long, narrow, and steep depressions on the nor too hot. Water vapour also contributes to
abyssal plain. The deeper trenches (> 5500 the stability and instability of the atmosphere.
metres) are called deeps. Reference: NCERT, Class 11th, Fundamentals
R

Option C is incorrect: Canyons are deep of Physical Geography, Chapter 7


E

concave gorges on the continental shelf, slope,


or rise. 99. Ans: B
T

Option D is incorrect: Abyssal plains, or deep Exp:


Wegener, in his ‘Continental Drift Theory,
A

sea plains, are undulating plains 2-3 miles


below sea level that cover 2/3rd of the ocean suggested that the movement responsible for
M

floor. Lying generally between the foot of a the drifting of the continents was caused by
continental rise and a mid-ocean ridge, abyssal pole-fleeing and tidal forces. The polar-fleeing
C
S
P

https://t.me/visioniastestseries2024
29

force is related to the rotation of the earth.


The earth is not a perfect sphere, as it has a
bulge at the equator. This bulge is due to the Pair 3 is correctly matched: The South Pacific
rotation of the earth. The second force Sea Route connects Western Europe and
suggested by Wegener was the tidal force due North America with Australia, New Zealand
to the attraction of the moon and the sun that and the scattered Pacific islands via the Panama
develops tides in oceanic waters. Wegener Canal. This route is also used for reaching Hong
believed that these forces would become Kong, the Philippines and Indonesia. The
effective over many millions of years. distance covered between Panama and Sydney
However, these forces were considered totally is 12,000 km. Honolulu is an important port on
inadequate by most of the scholars. this route.
Reference: NCERT, Class 11th, Fundamentals
of Physical Geography, Chapter 4

100. Ans: A
Exp:

Pair 1 is incorrectly matched: The Suez Canal


was constructed in 1869 in Egypt between Port
Said in the north and Port Suez in the south
linking the Mediterranean Sea and the Red
Sea. It gave Europe a new gateway to the Indian
Ocean and reduces the direct sea route distance
between Liverpool and Colombo compared to
the Cape of Good Hope route. It is a sea-level Reference: NCERT, Class 12th, Fundamentals
canal without locks which is about 160 km and of Physical Geography, Chapter 7
11 to 15 m deep.

Pair 2 is incorrectly matched: The Panama


Canal connects the Atlantic Ocean in the east
to the Pacific Ocean in the west. It has been

E
constructed across the Panama Isthmus

IN
between Panama City and Colon by the U.S.
government, which purchased 8 km of area on
either side and named it the Canal Zone.
L
N
.O
L
IA
R
E
T
A
M
C
S
P

https://t.me/visioniastestseries2024

You might also like